Chapter 28: Complementary and Integrative Health Question Bank

अब Quizwiz के साथ अपने होमवर्क और परीक्षाओं को एस करें!

allopathic=

biomedicine

chemical compounds that are ingested with the hope of achieving a therapeutic goal

botanical agents (herbs) and nutritional supplements

Ayurvedic medicine has its roots in which of the following areas? a. Japan b. China c. India d. Southeast Asia

c. India Ayurvedic medicine has its roots in India.

Ataxia is caused by

cerebral palsy, medication misuse, stroke or tumor along the cerebellum, Multiple sclerosis

Why is Zen meditation effective for a client with generalized anxiety disorder? a. Helps client forget issue causing stress b. Used to reduce hyperactivity in group therapy c. Includes yoga exercise for stress relief d. Assists client to enter state of calmness

d

Joints structure and functions

limits JOINT ROM

_________ include nondigestable food ingredients such as dietary fiber the enhance or maintain the growth of probiotic intestinal bacteria.

prebiotics

_________ microorganisms that exert beneficial health effects, such as Lactobacillus acidophilus and Bifidobacterium infantis; can lower frequency or duration of diarrhea

probiotics

Involves significant meaning in all of life, including illness and death

spirituality

how can one prepare the mind and body before beginning an imagery session?

using a relaxation technique

When obtaining information for a database, which of the following represents a nurse commitment and interest in reflected integrative medicine (CAM)?

"Do you take any vitamins or minerals, and if so, what?"

A nurse is assisting with teaching a client about homeopathy. Which of the following client statements indicates an understanding of the teaching?

"Homeopathy believes a substance that can cause a disease can also cure it."

Eastern Philosophy

- Using the Mind to Heal the Body.

•Colonics

-procedure for washing inner walls of colon by filling it with water or herbal solutions and then draining it

A nurse is teaching a client about food choices that promote immunity and healing. Which of the following foods is the best source of vitamin C?

1 cup strawberries

Herbal Medicine

80% of world population.Many prescription drugs derived from plants

Detoxification

Belief that physical impurities and toxins must be cleared

A client is extremely anxious while awaiting the results of a biopsy. What action by the nurse will assist the client with progressive relaxation techniques?

Have the client tighten and release different muscle groups.

A home care nurse delivers care that incorporates a philosophy that focuses on connections and interactions between parts of the whole. Which term best describes this philosophy? a) Complementary therapy b) Holism c) Homeopathy d) Integrative care

Holism Explanation: Holism is a theory and philosophy that focuses on connections and interactions between parts of the whole. Complementary therapy can be used to complement traditional medical interventions. Integrative care often incorporates sharing the responsibility in coordinating the best possible treatment plan for a client, including the client's choices for care and the provider's expertise. Homeopathy is based on the belief of supporting the body while the symptoms are allowed to "run their course."

Healing the person as a whole is a goal of what type of nursing?

Holistic

Nutritional Therapy

Includes supplements, vitamins, minerals, enzymes, fish oils

Spirituality and medicine are inseparable.

Native American Healing

Tremors causes

Parkinsons, Medication side effects and alcohol or drug withdraw

Naturopathy treatment

Restoration of health and normal body functions and Customized to client •

Traditional Chinese Medicine & Herbalism -

Using the power of herbs and energy practice to enhance overall wellness.

Ethnocentrism has prevented

Western health practitioners from learning new ways to promote health and prevent illnesses

Ethnocentrism

a healing modalitiy with the Assumption that one's own culture or ethnic group is superior to others

The nurse is certified in providing Therapeutic Touch and is preparing to initiate this for a client. What activity should the nurse perform initially? a. Requesting the client's permission to touch the body b. Moving the hands 2 to 6 in (5 to 16 cm) away from the client's skin surface c. Embracing the client for energy d. Calling to rebalance the soul

a. Requesting the client's permission to touch the body Prior to touching the client, the nurse should request permission and explain the process of therapeutic touch. All other interventions would be performed after this occurs.

According to survey results, who are the most prevalent users of complementary or alternative therapies? a. Women, ages 35-50, with college degree, former smokers b. Men, ages 50-65, with some high school, who smoke c. Women, ages 25-40, with high school degree, who smoke d. Men, ages 30-45, with college degree, who recently stopped smoking

a. Women, ages 35-50, with college degree, former smokers The most prevalent users of CAT are women, ages 35-50, with higher levels of education, who are former smokers.

Alignment

abnormal spinal curvature and cannot maintain alignment independently

addressed a person's qi -- Qi or chi in Japanese -- is believed to flow vertically in the body through an intricate structure of 12 primary meridians, energy circuits that nourish and support all cells and organs of the body

acupuncture

vague; luminous; glow surrounding something that may be containing electromagnetic field

aura

The nurse is certified in providing Therapeutic Touch and is preparing to initiate this for a client. What activity should the nurse perform first? a. Calling to rebalance the soul b. Requesting the client's permission to touch the body c. Embracing the client for energy d. Moving the hands 2 to 6 in (5 to 16 cm) away from the client's skin surface

b

A nurse working in a long-term care facility incorporates aromatherapy into her practice. For which patient would this nurse use the herb ginger? a) A patient who has insomnia b) A patient who has nausea c) A patient who has dementia d) A patient who has migraine headaches

b. Commonly used essential oils in a health care setting are ginger or peppermint for nausea and lavender or chamomile for insomnia.

integrative health care

A combination of complementary and alternative therapy with traditional medicine to facilitate healing

While assessing a client, the client tells the nurse that he is a follower of traditional Chinese medicine and the concept of qi. Based on the nurse's understanding of this concept, which treatment modality would the nurse expect the client to mention?

Acupuncture

Yoga types

Ashtanga, Bikram, Iyengar, Kripaula

Understanding one's dosha, or basic constitution, is central to which type of whole medical system? a) Ayurveda b) Traditional Chinese medicine c) Chiropractic medicine d) Naturopathy

Ayurveda Explanation: Central to Ayurvedic medicine is understanding the client's basic constitution, or dosha. People can display one dosha predominantly or can be a combination of two or three doshas.

• Emotionally healthy people Ayuverda

Balanced emotional states, sense of well-being or happiness

What is an essential component of holistic practice?

CHA

Prayer

Communication and fellowship with Deity or Creator • Self-care strategy

Ayurvedic medicine has its roots in which of the following areas? • China • Japan • Southeast Asia • India

Correct response: • India Explanation: Ayurvedic medicine has its roots in India.

A client comes to a clinic for a checkup. The clinic practices integrative care. During the visit, the client reports a "runny nose" accompanied by sinus pressure, cough and mild fever. The client states, "I think I have a cold." Which intervention would the nurse include for this client as part of integrative care? Select all that apply. • Using an over-the-counter nasal decongestant spray • Drinking lots of fluids throughout the day • Undergoing acupressure to appropriate sites for nasal congestion relief • Practicing good hand washing to prevent the risk for transmission • Using echinacea or zinc for prevention of colds

Correct response: • Using echinacea or zinc for prevention of colds • Undergoing acupressure to appropriate sites for nasal congestion relief • Using an over-the-counter nasal decongestant spray • Drinking lots of fluids throughout the day • Practicing good hand washing to prevent the risk for transmission Explanation: Integrative care uses some combination of allopathic (traditional medicine) and complementary health approaches. Therefore, using herbs, acupressure, nasal decongestants, fluids, and good hand washing would all be included.

On the advice of friends, a client on a palliative care unit has requested acupuncture. What it is the goal of this form of CAM? • allowing accumulated toxins to be released from the body • altering the client's perception and acceptance of reality • reconnecting the client's body, spirit, and emotions • restoring a healthy flow of energy along the meridians of the body

Correct response: • restoring a healthy flow of energy along the meridians of the body Explanation: Acupuncture either increases or decreases the flow of qi along the meridians of the body. It does not directly address the relationships between body, spirit, and emotions, nor does it aim to release toxins or change an individual's perception of reality.

Latin american and Latinos

Curanderismo

A client asks the nurse about the use of healing touch. Which statement regarding healing touch is accurate? A) Healing touch is too expensive for most clients. B) Aromatherapy is used in healing touch. C) Supplemental music is used during healing touch D) Healing touch does not use injections

D) Healing touch does not use injections

Hypertrophy

Enlargement of muscle tissue

Functional Medicine

Helping people at a functional - not disease level.

What are the three categories of complementary health approaches?

Mind body practices, natural products and whole or alternative medical systems

Which principle does not encompass the basic goals of integrative medicine?

Reject allopathic medicine and embrace CAM practices.

REDUCES ANXIETY, REDUCES MUSCLE TENSION AND PAIN IMPROVES FUNCTION OF IMMUNE SYSTEM ENHANCES SLEEP AND REST IMPROVES SENSE OF WELL-BEING

Relaxation

What is another word for hypertonia?

Spasticity

Ayurvedic Medicine

The Science of Life.

What is health in allopathic medicine?

The absence of disease

What does remedies that stimulate self- healing capacity means?

The practice of medicine that embraces holistic and natural approach to the treatment of the sick

What is hydrotherapy?

Use of water as a healing treatment • Uses body's response to heat and cold

Traditional Naturopathy -

Using "Right Living" to Rebalance the Body

A client is following a Mediterranean diet to increase heart health. The nurse would offer which items as a snack? Select all that apply.

Whole grain crackers Fruit Carrots with humus

Shamanism

a belief that illness originates in the spirit world and usually involves a loss of power; treatment consists of first, restoring the individual's power, and second, treating symptoms

A client is asking for the nurse to explain acupuncture. What would the nurse tell the client? a. Acupuncture is used to correct disharmony. b. Acupuncture is only done in Eastern countries. c. Acupuncture is beneficial to creating a mood of distraction. d. Acupuncture is a dangerous option for the treatment of disease.

a. Acupuncture is used to correct disharmony. Acupuncture can be used to correct disharmony or prevent disharmony from developing.

Which of the following has been known to achieve benefits for clients with cancer through the use of the mind to visualize a positive physiologic effect? a. Imagery b. Hypnosis c. Humor d. Biofeedback

a. Imagery Imagery is a physiologic technique that uses the mind to visualize a positive physiologic effect. When using imagery, clients conjure up mental images of their body waging and winning a battle with the disease process. Hypnosis is a therapeutic intervention that facilitates a physiologic change through the power of suggestion. Humor would not be appropriate in this situation. Biofeedback is a technique in which a person voluntarily controls one or more physiologic functions.

Paralysis

absence of strength secondary to nervous impairment

A client suffers from chronic pain. The nurse suggests the client have monthly massages. This is an example of:

alternative medicine.

moxibustion

ancient Chinese method of treatment that uses a powdered plant substance on the skin to raise a blister.

Ayurvedic medicine has its roots in which of the following areas? a. China b. India c. Japan d. Southeast Asia

b

A nurse is interviewing a client who has come to the clinic for an evaluation. During the visit, the client tells the nurse that she also sees a provider who practices traditional Chinese medicine. The client describes treatment using acupuncture with the burning of an herb above the acupuncture needle. The nurse interprets this as: a. healing touch. b. moxibustion. c. shamanism. d. homeopathy.

b. moxibustion. Moxibustion is the burning of an herb above an acupuncture needle and is associated with traditional Chinese medicine. Healing touch involves the placing of hands on or near the body in patterns. Homeopathy believes that when symptoms are suppressed, such as with medication, the condition penetrates deeper into the body making it more difficult to cure. Shamanism involves the belief that illness and other forms of distress are thought to originate in the spirit world.

Dosha

body type and dosha imbalance

A client suffers from chronic pain. The nurse suggests the client have monthly massages. This is an example of: a. adjuvant medicine. b. palliative medicine. c. alternative medicine. d. allopathic medicine.

c

When obtaining information for a database, which of the following represents a nurse commitment and interest in reflected integrative medicine (CAM)? a. "Which diseases do you suffer from and what are your allergies?" b. "Which prescription medications do you take daily?" c. "Do you take any vitamins or minerals, and if so, what?" d. "Which types of foods do you consume in 24 hours?"

c. "Do you take any vitamins or minerals, and if so, what?" Starting with the initial contact with a client, the nurse includes a basic assessment of the client's use of CAM in an intake interview. The nurse asks about vitamins and supplements as well as any healing practices.

A nurse is guiding a patient in the practice of meditation. Which teaching point is most useful in helping the patient to achieve a state of calmness, physical relaxation, and psychological balance? a) Teach the patient to always lie down in a comfortable position during meditation. b) Teach the patient to focus on multiple problems that the patient feels demand attention. c) Teach the patient to let distractions come and go naturally without judging them. d) Teach the patient to suppress distracting or wandering thoughts to maintain focus.

c. Meditators should have an open attitude by letting distractions come and go naturally without judging them. They should also maintain a specific, comfortable posture lying down, sitting, standing, walking, etc.; focus attention on a mantra, object, or breathing; and not suppress distracting or wandering thoughts; instead they should gently bring attention back to focus.

Lordosis

cervical and lumbar part of the spine that curves towards the front of the body

integrative health

combination of complementary health and conventional health approaches in a coordinated way

____ can be used with traditional medical interventions + and complement them

complementary therapy

Holism

connections and interactions between parts of a whole

based on a scientific approach for treating illness of injury

conventional medicine

Which of the following is an important aspect of Ayurveda? a. Originated in Asia. b. Mind plays an important part in healing the body. c. Meditation is an integral part. d. Balancing of the doshas.

d

Which would be the least consistent with the Native American/Canadian Indigenous view of disease? a. Disharmony with Mother Earth b. A result of negative thinking c. Violation of a taboo d. Balance of yin and yang

d

Chorea causes

degenerative muscles cells of the brain, drugs, toxins and inflammmation of body system

Iyengar Yoga

focuses on proper alignment and holding poses for a relatively longer time

complementary health approaches

interventions that can be used with conventional medical interventions

Rheumatoid Arthritis

joints deformities, swollen and erythema

Athetosis

slow, involuntary, non repetitive, reviving movements occurs in different parts of body such as in fingers, hands, toes and tongues and neck at different times

Ancient form of healing

sound healing

aromatherapy

the use of essential oils of plants to treat symptoms

Allopathic health

traditional medial care

CAT: biologically based practices

using herbs, vitamins, minerals, probiotics, etc

Muscle Mass Atrophy

wasting or loss of muscle tissue

hemiparesis

weakness on one side of the body

when is the imaginary situatiuon more likely to take place?

when all senses are involved because the imagery situation is fully encoded in the body

CAT: consist of a set of beliefs about the origin of diseases, ways to promote health, and types of treatment

whole (or alternative) medical sysems

In prenatal classes, the nurse teaches pregnant clients to use meditation during labor contractions to ease the pain. What does the nurse explain to the pregnant clients are the elements of meditation? Select all that apply. a) Closed attitude b) Massaging abdomen c) Quiet environment d) Comfortable position e) Focus of attention

• Comfortable position • Quiet environment • Focus of attention Explanation: There are four elements common to most types of meditation: comfortable position, quiet environment, focus of attention, and open attitude. Closed attitude and massaging abdomen are not elements of medication

Major concerns of nutritional therapy

• Efficacy • Consistency • Safety

A nurse attempts to integrate complementary and alternative therapies (CAT) into nursing practice. Which of the following are basic principles of CAT? Select all that apply. a) Illness occurs in either the mind or the body, which are separate entities. b) Health is the absence of disease. c) Health is a state characterized by a dynamic balance of mind, body, and spirit. d) Illness is a manifestation of imbalance or disharmony and is a process. e) Curing is accomplished by external agents.

• Health is a state characterized by a dynamic balance of mind, body, and spirit. • Illness is a manifestation of imbalance or disharmony and is a process. Explanation: In CAT, health is a state characterized by a dynamic balance of mind, body, and spirit; illness is a manifestation of imbalance or disharmony and is a process. In this system, healing is a natural, slow process that involves the body, mind, and spirit.

tradional chinese medicine

• Imbalance or interruption in the flow of qi • Mind, body, spirit, and emotion are never separated.

Excercise program in ayuverda

• Yoga • Breathing exercises • Meditative techniques

A nurse is assisting with providing an in-service for staff members about complementary and integrative health. Which of the following statements by a staff member indicates an understanding of the teaching?

"Scented oils might decrease discomfort."

Orthomolecular Medicine

- The Right molecules in the right amounts.

Bioregulatory Medicine -

- The Science of Self-Healing

• Kripaula, or "gentle yoga":

: focuses on relaxation and coming into balance

A nurse is assisting a massage therapist to identify the best client for massage therapy. Which of the following clients should the nurse recommend?

A client who has postpartum depression

A nurse is caring for a client who has a sunburn. The nurse should identify that which of the following herbal supplements is used to treat sunburns?

Aloe vera

What is faith and prayer?

Beliefs and expectations about life, ourselves, and others • Belief in a Supreme Being

What do most holistic nurses use for self care?

CHA or tea

While assessing a client, the client tells the nurse that he is a follower of traditional Chinese medicine and the concept of qi. Based on the nurse's understanding of this concept, which treatment modality would the nurse expect the client to mention? • Physiotherapy • Allopathy • Acupuncture • Therapeutic Touch (TT)

Correct response: • Acupuncture Explanation: Acupuncture is based on energy regulation of qi through meridians. It is a core principle in traditional Chinese medicine. Therapeutic Touch (TT) is not a form of traditional Chinese medicine and is based on the consciously directed process of energy exchange. Physiotherapy and allopathy are not based on the concept of qi.

A nurse who "unblocks" and "clears" congested areas of energy in a client's body to promote comfort is applying the phenomenon known as: • Tactile manipulation • Therapeutic Touch (TT) • "Unruffling" touch • Interpersonal touch

Correct response: • Therapeutic Touch (TT) Explanation: Therapeutic Touch (TT) involves "unruffling," or clearing, congested areas of energy in the body and redirecting this energy. After assessing a client's "energy field," the nurse uses therapeutic touch to promote comfort, relaxation, healing, and a sense of well-being.

A nurse who "unblocks" and "clears" congested areas of energy in a client's body to promote comfort is applying the phenomenon known as: A) "Unruffling" touch B) Interpersonal touch C) Tactile manipulation D) Therapeutic Touch (TT)

D) Therapeutic Touch (TT)

used to increase energy; and can result in elevation of blood pressure

Ginseng

Healers use medicine objects and ceremonial treatments in

Native american healing

Physically healthy people in ayurveda

Proper functioning of senses, digestion, elimination

What is integrative health?

Referes to the combination of complementary health and convential health approaches in a coordiated way

Animal-assisted therapy

Specifically selected animals used as a treatment modality in health and human service settings

consists of placing very thin, short, sterile needles at particular acupoints, believe to be centers of nerver and vascular tissue, along a meridian.

acupuncture

Do no injest what?

aromatherapy

uses essential oils

aromatherapy

While assessing a client, the client tells the nurse that he is a follower of traditional Chinese medicine and the concept of qi. Based on the nurse's understanding of this concept, which treatment modality would the nurse expect the client to mention? a. Physiotherapy b. Acupuncture c. Allopathy d. Therapeutic Touch (TT)

b. Acupuncture Acupuncture is based on energy regulation of qi through meridians. It is a core principle in traditional Chinese medicine. Therapeutic Touch (TT) is not a form of traditional Chinese medicine and is based on the consciously directed process of energy exchange. Physiotherapy and allopathy are not based on the concept of qi.

A nurse is caring for a client admitted for a prolonged stay on a medical-surgical unit. The client has been having difficulty sleeping and appears depressed. Applying the holistic health model, which action taken by the nurse would be most appropriate? a. Document the observations, and encourage the client that he or she will feel better once going home b. Ask the client's family to bring some items from home, such as a blanket, pillow, or pictures c. Suggest the client try some meditation and contact the health care provider to request medication to help with sleep d. Reassure the client how lucky he or she is to have family visit often and reassure the client that he or she will be going home soon

b. Ask the client's family to bring some items from home, such as a blanket, pillow, or pictures Holism is a theory and philosophy that focuses on connections and interactions between parts of the whole, as well as the body's interaction with the environment. Holism recognizes that a person's environment and habits are part of who the person is. The holistic model is not necessarily a counterpoint to the biomedical model. While a focus on the mind is congruent with holism, this does not preclude treatment of bodily symptoms. Reassurance and encouragement may seem necessary but are not components in holistic care and can often feel ingenuine to clients. Suggesting meditation may help the client but is not the most appropriate measure.

A nurse can best help a client who is undergoing chemotherapy and using guided imagery with this by doing which of the following? a. Explaining to the client that it is not a good idea to record their own imagery tape. b. Assisting the client to find an appropriate imagery tape to use. c. Helping the client learn about the different poses that can be performed. d. Promoting the client's use of imagery only after a stressful event occurs.

b. Assisting the client to find an appropriate imagery tape to use. Clients use imagery to help with relaxation and this can use all five senses. Many times, clients use a guided imagery tape that includes a script to help with imagery; clients also can record their own tape. Clients do not necessarily use poses with this; this would be more related to use of yoga. Guided imagery can help both during and after a stressful event to help the client relax.

why are some consumers and practitioners attracts to herbs?

because they are "natural" plant products which are perceived as more compatiable with the body than manufactured phamaceutical agents.

During a client interview, the client tells the nurse about using ginkgo biloba to improve memory. When reviewing the client's medication history, which medication if used by the client would be a cause for concern? a. Guaifenesin b. Acetaminophen c. Warfarin d. Azithromycin

c. Warfarin Ginkgo biloba, the most widely sold herb in Europe and used by many to improve memory, affects platelet function and thus should not be used concurrently with warfarin or aspirin. It apperars that acetaminophen, guaifenesin, and azithromycin would not be a concern.

____ are concentrated areas of energy aligned vertically in the body that relate to each other as well as to specific areas of the body, mind, and spirit

chakras

A nurse is caring for a hospitalized client who states: "I feel so sick all the time; my aura must be disturbed by all of these bad force fields." What is an appropriate NANDA-I diagnosis for this client? a. Social isolation b. Impaired coping c. Hopelessness d. Disturbed energy field

d

A nurse who "unblocks" and "clears" congested areas of energy in a client's body to promote comfort is applying the phenomenon known as: a. "Unruffling" touch b. Interpersonal touch c. Tactile manipulation d. Therapeutic Touch (TT)

d

Which nurse theorist developed Therapeutic Touch (TT)? a. Dorothea Orem b. Martha Rogers c. Jean Watson d. Dolores Krieger

d

The nurse is caring for a client diagnosed with pancreatic cancer. The client would like to use integrative care as a health care choice. What is the best response by the nurse? a. "The health care provider will make a plan for your care and inform you what that will be. It is not advisable for you to participate in any type of therapy outside of this plan." b. "You should consider using dietary supplements and botanicals, because this approach to your care can prove to have a healing effect on the body." c. "It is not proven that complementary health approaches have any value in treating terminal illnesses such as pancreatic cancer. It is best to avoid those types of therapies." d. "There are several alternative medicine modalities that could help with pain and relaxation during your treatment. I will get you some information on them."

d. "There are several alternative medicine modalities that could help with pain and relaxation during your treatment. I will get you some information on them." A person who uses integrative care uses some combination of allopathic medicine and complementary therapy approach. This integrated process gives the client the best opportunity for therapeutic effective care. Complementary therapy is a valid and research-based alternative to traditional medicine and can be a very effective option in conjunction with traditional medicine. Dietary supplements and botanicals may be beneficial, but the nurse should be cautious in suggesting this without the health care provider's approval, becauses some botanicals and herbal supplements can interact with other medications, nor will they "heal" the client. The health care provider and the client will form a plan of care together.

Which would be the least consistent with the Native American/Canadian Indigenous view of disease? a. Violation of a taboo b. Disharmony with Mother Earth c. A result of negative thinking d. Balance of yin and yang

d. Balance of yin and yang The Native American/Canadian Indigenous view disease as a disharmony with Mother Earth, a result of negative thinking, or violation of a taboo. Chinese medicine proposes that health is the outcome of balancing yin and yang.

A nurse practitioner is conducting a presentation at a local community center about complementary health approaches. One of the participants asks the nurse practitioner, "Everybody is talking about relaxation. Just how does relaxation help a person?" The nurse responds, integrating which effect as being associated with relaxation? Select all that apply. a. Lowered immune response b. Improved sense of well-being c. Reduced muscle tension d. Better sleep and rest e. Less anxiety

d. Better sleep and rest e. Less anxiety c. Reduced muscle tension b. Improved sense of well-being Explanation: Relaxation can be useful whether a client is experiencing a single stressful event, such as surgery, or chronic stress. Client benefits include reduced anxiety, reduced muscle tension and pain, improved functioning of the immune system, enhanced sleep and rest, and an improved overall sense of well-being.

An occupational therapist conducts daily relaxation exercises with clients who are receiving treatment on the psychiatric unit of a hospital. Stimulation of which of the following components of the nervous system will result in relaxation for the clients? a. Autonomic nervous system b. Central nervous system c. Sympathetic nervous system d. Parasympathetic nervous system

d. Parasympathetic nervous system Stimulation of the parasympathetic nervous system results in an alleviation of stress. Direct stimulation of the sympathetic nervous system increases stress. The autonomic nervous system encompasses both the sympathetic and parasympathetic divisions. The central nervous system consists of the brain and spinal cord; neither is stimulated to facilitate relaxation.

A client suffers from chronic pain. The nurse suggests the client have monthly massages. This is an example of: a. palliative medicine. b. adjuvant medicine. c. allopathic medicine. d. alternative medicine.

d. alternative medicine. The use of conventional therapy as seen with CAM includes the use of herbal medicine, massage, megavitamins, self-help groups, folk remedies, energy healing, and homeopathy.

What are the curing seeks in allopathic medicine?

destroy the invading organism or to repair the affected part

Spiritual Therapy

faith and prayer

guided imagery

using all five senses to imagine an event or body process unfolding according to a plan, focusing on evoking pleasant images to replace negative or stressful feelings and to promote relaxation

CAT: manipulative and body based practices

working with the bones, joins, soft tissues, and body systems

The nurse is educating a client on how to use herbs and supplements as part of an integrated treatment plan. Which teaching points would the nurse include? Select all that apply. a) Whenever possible, buy products with more than one ingredient. b) Take a higher than recommended dose of herbs to initiate the therapeutic effect. c) Be knowledgeable about the product and its therapeutic actions. d) Buy herbs and supplements that are standardized. e) Give the product adequate time to work. f) Use the Internet to buy herbs and supplements.

• Be knowledgeable about the product and its therapeutic actions. • Buy herbs and supplements that are standardized. • Give the product adequate time to work. Explanation: In the United States, FDA classifies herbs as a food and not a drug. This is important because herbs are not regulated for quality and potency as drugs are, and herbal formulas can be sold without being studied to see if they are helpful or harmful. For these reasons, herbs that have a certain degree of standardization should be purchased. There needs to be adequate time given for levels to accumulate in the client's system. Some herbs are either contraindicated or cause a synergistic effect when ingested with certain medications.

The nurse is caring for a client diagnosed with pancreatic cancer. The client would like to use integrative care as a health care choice. What is the best response by the nurse?

"There are several alternative medicine modalities that could help with pain and relaxation during your treatment. I will get you some information on them."

Bikram Yoga

: is done in a 105-degree studio with 26 set postures

Akinesia vs hypokinesia

Absence of movement vs reduce or slower movements

What is holistic perspective?

All living organisms including humans are continuously connecting and interacting with the environment

A nurse is reading an online journal article about different approaches to health. The nurse is reading about a practice approach that is supported by evidence-based practice and is particularly effective when aggressive treatment is needed in an emergency situation. The nurse is reading about which type of approach?

Allopathic

What is another term for conventional medical interventions?

Allopathic medicine

What is a sign or signs of lack of endurance?

Alterations in vitals such as hypertensive, increase in HR, RR, generalized muscle weakness , pale and dizzy

A nurse can best help a client who is undergoing chemotherapy and using guided imagery with this by doing which of the following?

Assisting the client to find an appropriate imagery tape to use.

Every animal, plant, and mineral has an electromagnetic field •

Bioelectromagnetic

Magnetic fields penetrate body, affect functioning of cells, tissues, organs, systems.

Bioelectromagnetic

The nurse is educating a client on how to use herbs and supplements as part of an integrated treatment plan. Which teaching points would the nurse include? Select all that apply.

Buy herbs and supplements that are standardized. Give the product adequate time to work. Be knowledgeable about the product and its therapeutic actions.

What are the three cautions with herbal medicine?

Caution when combining with prescription and over-the-counter medications Caution about becoming dependent on herbal remedies Caution pregnant and breast-feeding women not to take herbs

• Unconditionally loving • Opportunities for affection • Achievement of trust • Responsibility • Empathy towards others • Reason to get up in the morning • Source of reassurance

Charcteristics of companion animals in animal assisted therapy

Preventive maintenance to ensure problem does not recur is apart of

Chiropractic Health Care

Allopathic medicine is less effective in the care of clients with:

Chronic illnesses.

In prenatal classes, the nurse teaches pregnant clients to use meditation during labor contractions to ease the pain. Which elements of meditation are important for the nurse to emphasize? Select all that apply.

Comfortable position Quiet environment Focus of attention

Nervous System Physcial Assessment

Communication between neurons, reflect and response to stimuli, volitional and intentional movements, vital sense intact and Equilibrium for ADL.

A chant is a form of which category of meditation?

Concentrative

When obtaining information for a database, which of the following represents a nurse commitment and interest in reflected integrative medicine (CAM)? • "Which types of foods do you consume in 24 hours?" • "Which diseases do you suffer from and what are your allergies?" • "Do you take any vitamins or minerals, and if so, what?" "Which prescription medications do you take daily?"

Correct response: • "Do you take any vitamins or minerals, and if so, what?" Explanation: Starting with the initial contact with a client, the nurse includes a basic assessment of the client's use of CAM in an intake interview. The nurse asks about vitamins and supplements as well as any healing practices.

A client is asking for the nurse to explain acupuncture. What would the nurse tell the client? • Acupuncture is beneficial to creating a mood of distraction. • Acupuncture is only done in Eastern countries. • Acupuncture is used to correct disharmony. • Acupuncture is a dangerous option for the treatment of disease.

Correct response: • Acupuncture is used to correct disharmony. Explanation: Acupuncture can be used to correct disharmony or prevent disharmony from developing.

A nurse is providing care to a client who has come to the outclient clinic for chemotherapy. The client tells the nurse that to cope with the stress of chemotherapy, he uses a technique in which he "goes to my happy place, the beach, and I picture myself lying there under the warm sun, with the sound of the waves lapping at the shore." The nurse interprets this as which technique? • Yoga • Tai chi • Meditation • Guided imagery

Correct response: • Guided imagery Explanation: Guided imagery focuses on evoking pleasant images to replace negative or stressful feelings and to promote relaxation. It involves using all five senses to imagine an event or body process unfolding according to a plan. During a painful or stressful event, the client can "go to a favorite place" and imagine being there with all the pleasant experiences related to that space. Meditation refers to a group of techniques in which the person learns to focus attention. Tai chi is a martial art, mind-body practice that involves physical movement, mental focus, deep breathing and relaxation. Yoga is a mind-body practice that involves the combination of physical movements, breathing practices, and relaxation techniques.

The nurse is caring for a client whose treatment has been based on the Ayurveda medical system. Which nursing intervention incorporates this client's beliefs into the nursing plan? • Basing practice on the yin-yang theory • Including the client's shaman in the plan of care • Helping the client to balance his or her dosha • Preparing the client for exercises that help him or her regulate qi

Correct response: • Helping the client to balance his or her dosha Explanation: Central to Ayurvedic medicine is an understanding of th client's basic constitution, or dosha. The three doshas in Ayurvedic medicine are vata (changeable), pitta (intense), and kapha (relaxed). Yin-yang and qi are associated with traditional Chinese medicine. A shaman is associated with shamanism (involves the spirit world).

Which of these assessment findings indicate a positive outcome for a client after acupuncture? • The client has lost 8 lb (3.63 kg) over the last 2 months. • The client has increased his or her flexibility and strength. • The client reports a reduction in pain level to 3 out of 10. • The client has improved balance and coordination.

Correct response: • The client reports a reduction in pain level to 3 out of 10. Explanation: Acupuncture can help with reduction of pain and nausea with clients. It is not a treatment for balance and coordination, strength and flexibility, or weight loss.

What is hydrotherapy used to do?

Decrease pain and fever • Reduce swelling and cramps • Induce sleep • Improve physical and mental tone

What is hypertonia?

Increased muscle tone, tightness and inability to stretch

Ayurvedic medicine has its roots in which of the following areas?

India

Primary consideration • Least invasive method

Naturopathy

Homeopathy

Self-healing system • Law of similars • Remedies stimulate self healing capacity

Naturopathy

System of medicine and a way of life

what method is non invasive( injections, tubes. etc.), effective, non toxic and is economical (can be done anywhere and does not require equipment)

The healing touch

A nurse is caring for a client who is receiving palliative care. Which of the following non-pharmacological nursing interventions should the nurse provide the client?

Therapeutic touch

Acupuncture • Acupressure • Herbal medicine • Massage • Heat therapy • Qigong • T'ai chi

Traditonal Chinese Treatments or medicines

A client at the clinic is considering the use of acupuncture to assist with smoking cessation. When describing this practice, the nurse would explain that which concept is involved as the focus? a. Qi b. Yin c. Yang d. Alignment

a

are sensitive to culture and harmony between person and universe

alternative medicine

Which changes in diet would the nurse recommend for a client who is considering use of a holistic diet? Select all that apply. a. Increased intake of milk and cheese. b. Avoidance of processed food. c. Reduction in intake of soft drinks. d. Use of artificial sweetener instead of sugar. e. Use of a vegetarian diet.

b. Avoidance of processed food. e. Use of a vegetarian diet. c. Reduction in intake of soft drinks. Clients who use a holistic diet are bringing a more natural approach to eating. This involves avoidance of processed food and soft drinks, as well as reduction in the intake of dairy products, including milk and cheese. A person observing a vegetarian diet and eating natural foods without preservatives is also incorporating a holistic approach to choice of food.

A nurse who "unblocks" and "clears" congested areas of energy in a client's body to promote comfort is applying the phenomenon known as: a. "Unruffling" touch b. Therapeutic Touch (TT) c. Interpersonal touch d. Tactile manipulation

b. Therapeutic Touch (TT) Therapeutic Touch (TT) involves "unruffling," or clearing, congested areas of energy in the body and redirecting this energy. After assessing a client's "energy field," the nurse uses therapeutic touch to promote comfort, relaxation, healing, and a sense of well-being.

The client has been diagnosed with a disease and is seeking information about naturopathy. The nurse, explaining about naturopathy, encourages the client to do what? a. Exercise at least once per week. b. Include well-cooked processed foods in diet. c. Obtain adequate sleep each night. d. Concentrate on the implications of the disease.

c

The practitioner is working with structures and systems within the human body, including the bones, muscles, joints, and circulatory system. Which one of the complementary and alternative therapy domains is being used? a. Mind and body medicine b. Energy medicine c. Manipulative and body-based practice d. Biologically based practice

c

The nurse in a medical unit is collecting a client's history and asks the client about the use of complementary and alternative therapies. The client asks why the nurse needs to know about this. What is the nurse's best response? a. "I want to make sure you understand all the risks of these treatments." b. "I am just curious on what types of treatments are used by people." c. "It's important that we list all your home care needs for the hospital." d. "It will help me so that I can recommend use of these for other clients."

c. "It's important that we list all your home care needs for the hospital." Nurses need to include complementary and alternative therapy in assessment of the client's current treatments to ensure an understanding of the safety and effectiveness of the treatments, particularly if the client is also receiving allopathic treatment. The nurse would want to be positive with this questioning so that the client is not defensive regarding any complementary and alternative treatments.

On the advice of friends, a client on a palliative care unit has requested acupuncture. What it is the goal of this form of CAM? a. altering the client's perception and acceptance of reality b. allowing accumulated toxins to be released from the body c. restoring a healthy flow of energy along the meridians of the body d. reconnecting the client's body, spirit, and emotions

c. restoring a healthy flow of energy along the meridians of the body Acupuncture either increases or decreases the flow of qi along the meridians of the body. It does not directly address the relationships between body, spirit, and emotions, nor does it aim to release toxins or change an individual's perception of reality.

____ is a health care profession that focuses on the relationship between the body's structure-mainly the spine- and its functioning

chiropractic medicine

A client asks the nurse about taking herbal medications. Which of these is the nurse's best response regarding safety of the herbal medications? a. "You can take combination products to save money on these medications." b. "Herbal medications are natural products so there are not side effects with these." c. "Your effects from herbal medications are quicker than with traditional medicine." d. "Name brand products with herbal medications usually are of higher quality."

d

In prenatal classes, the nurse teaches pregnant clients to use meditation during labor contractions to ease the pain. Which elements of meditation are important for the nurse to emphasize? Select all that apply. a. Massaging abdomen b. Closed attitude c. Focus of attention d. Comfortable position e. Quiet environment

d. Comfortable position e. Quiet environment c. Focus of attention There are four elements common to most types of meditation: comfortable position, quiet environment, focus of attention, and open attitude. Closed attitude and massaging abdomen are not elements of meditation.

A nurse is teaching a client experiencing stress about how relaxation helps to reduce the effects of stress on the body. Which underlying concept would the nurse integrate into the explanation about how relaxation works? a. Slows circulation throughout the body b. Increases the body's natural immunity c. Activates natural pleasure centers d. Helps to increase the effects of parasympathetic nervous system on the mind and body

d. Helps to increase the effects of parasympathetic nervous system on the mind and body Relaxation techniques promote parasympathetic nervous system activity, helping to reduce sympathetic activity and restore the balance of the two systems. The ultimate goal is to increase the parasympathetic system influence in the mind-body and thus reduce the effect of stress and stress-related illness on the body. Natural products can boost the body's immunity. Energy healing activates natural pleasure centers. Hands-on techniques such as massage promote circulation throughout the body.

A nurse is caring for a postoperative patient who is experiencing pain. Which CHA might the nurse use to ensure active participation by the patient to achieve effective pre- or postoperative pain control? a) Acupuncture b) TT c) Botanical supplements d) Guided imagery

d. Imagery involves using all five senses to imagine an event or body process unfolding according to a plan. A patient can be encouraged to "go to a favorite place." With the other modalities, the patient is more passive.

A nurse is applying healing touch to a postoperative client's wound. What benefit of healing touch should the nurse emphasize to the client's family? a. It will decrease the need for fluids after surgery. b. It will improve the nurse-client relationship. c. It will enhance family relationships. d. It will help the wound heal.

d. It will help the wound heal. Healing touch uses a collection of energy techniques to assess and treat the human energy system. It can be used for a variety of purposes, including stimulating wound healing. The healing touch will not assist with the nurse client relationship or enhance family relationships. Fluids are not affected by the healing touch on the wound healing process.

In Qi Gong and Tai chi, excercise is

dynamic and gental

Chelation therapy

introduction of chemicals into the bloodstream that bind with heavy metals in the body

Movement Alteration can be

involuntary and due to an underlying neurological reason

Chorea

involuntary, unpredicatable, irregular muscle movements- no control at all

Scoliosis

is a postural deformity/curvature deformity of spine that can lead to difficulty breathing etc.

Tremors

is the rhythmic, uncontrollable muscle contractions, occurs continuous steady and does not change shaking movements in one or more parts of the body-

Hypermobility

joints stretch more than or farther normal- flexibility

Ataxia

lack of muscle control/coordination or movements with voluntary movements with picking up objects or walking

_________ stimulates the immune system by increasing the white blood cells counts and lowering cortisol

laughter

kava and comfrey

linked to serious liver damage

Why is endurance important?

patient may not be able to tolerate activity

_____ a bontanical oil; is helpful in decreasing nausea

peppermint

Cupping

placing a heated cup on the skin to create a slight suction

A patient with dermatitis has sought care from a homeopathic practitioner seeking treatment for dry, itchy, and inflamed skin. Which statement is characteristic of homeopathy?

"I'm going to recommend a diluted solution that is derived from poison ivy."

According to survey results, who are the most prevalent users of complementary or alternative therapies? A) Women, ages 35-50, with college degree, former smokers B) Men, ages 50-65, with some high school, who smoke C)Women, ages 25-40, with high school degree, who smoke D) Men, ages 30-45, with college degree, who recently stopped smoking

A) Women, ages 35-50, with college degree, former smokers

The nurse in a medical unit is collecting a client's history and asks the client about the use of complementary and alternative therapies. The client asks why the nurse needs to know about this. What is the nurse's best response? A) "It will help me so that I can recommend use of these for other clients." B) "I want to make sure you understand all the risks of these treatments." C) "It's important that we list all your home care needs for the hospital." D) "I am just curious on what types of treatments are used by people."

C) "It's important that we list all your home care needs for the hospital."

Humor and laughter is helpful with what three key points

Establishing relationships • Relieving tension and anxiety • Facilitating learning • Coping with painful feelings

Muscular Physical Assessment

FLUID MOVEMENTS, GAIT, MUSCLE MASS, TONE AND STRENGTH, GROSS AND FINE MOVEMENTS, STABLE CENTER OF GRAVITY, ENDURANCE IS ADEQUATE FOR NEEDS

Skeletal Physical Assessment

FULL JOINT ROM, STRAIGHT ALIGNMENT, PROPORTIONAL BONE STRUCTURE, POSTURE UP RIGHT AND AT EASE, STURDY FRAM WITH INTACT JOINTS

Spascity Hemiparesis

Gait-Posture- one side of the muscles of the body is in a constant state of contraction caused by some head injury

A client who is receiving chemotherapy and experiencing significant nausea asks the nurse about using aromatherapy to help alleviate the nausea. Which essential oil would the nurse most likely suggest to address the client's nausea?

Ginger

Mentally healthy people • Ayuverda

Good memory, comprehension, intelligence, reasoning ability

A nurse is assisting with teaching a class about using holistic nursing interventions to care for clients. Which of the following information should the nurse include?

Holistic nurses empower clients to actively participate in their own care.

What is complementary therapy ?

Implies a therapy used in conjunction with a conventional one

A nurse is researching a standardized herbal supplement to help a client with pain. What does the word standardized mean? a) It can be toxic in higher-than-normal doses. b) It is not contaminated by other ingredients. c) It contains a certain amount of an active ingredient. d) It is an all-natural product.

It contains a certain amount of an active ingredient. Explanation: A standardized product means that it contains a certain amount of active ingredient. Since this industry is not regulated, contamination and inconsistency in the amount of product sometimes occur.

• Do not provide emergency care or major surgery • Rarely prescribe drugs • Treat clients in private practice and outpatient clinics

Naturopathy

Which scenario is an example of tertiary prevention? a. Sam is prescribed an antihypertensive medication to control his newly diagnosed high blood pressure. b. Linda is having her first colonoscopy because she just turned 50. c. A school nurse is giving a talk about good handwashing to a first-grade class. d. A pamphlet on nutrition for children is given to all clients in a pediatric clinic. e. A hearing test is done on a newborn in the hospital.

a

horticultural therapy

gardening

what are echinacea and goldenseal (separate or in combo) commonly used for?

respiratory infections

healing tough (HT)

uses a collection of energy techniques to assess and treat the human energy system, thereby affecting physical, emotional, mental, and spiritual health and healing

A nurse is caring for a client who is scheduled for an emergency appendectomy and reports increasing anxiety. Which of the following statements should the nurse make to implement a complementary health strategy?

"Try to take slow, deep breaths in and out."

Emerging science that studies how living organisms interact with electromagnetic fields

Bioelectromagnetic

Naturopathy therapy

Consumption of specific diets or supplements to prevent or treat illnesses such as • Vitamins, minerals, amino acids, herbs, other botanical and miscellaneous substances

• Traditional healing system with Western biomedical beliefs, treatments, and practices

Curanderismo

A client asks the nurse about the use of healing touch. Which statement regarding healing touch is accurate?

Healing touch does not use injections.

The nurse is caring for a patient whose treatment has been based on the Ayurveda medical system. Which nursing intervention incorporates this patient's beliefs into the nursing plan? a) Including the patient's shaman in the plan of care b) Preparing the patient for exercises that help the patient regulate qi c) Basing practice on the yin-yang theory d) Helping the patient to balance his dosha

Helping the patient to balance his dosha Explanation: Central to Ayurvedic medicine is an understanding of th patient's basic constitution, or dosha. The three doshas in Ayurvedic medicine are vata (changeable), pitta (intense), and kapha (relaxed). Yin-yang and qi are associated with traditional Chinese medicine. A shaman is associated with shamanism (involves the spirit world)

Which of the following has been known to achieve benefits for clients with cancer through the use of the mind to visualize a positive physiologic effect?

Imagery

Ayurvedic medicine has its roots in which of the following areas? a) India b) China c) Southeast Asia d) Japan

India Explanation: Ayurvedic medicine has its roots in India.

Does the FDA regulate herbals?

No

What is hypotonia?

Not enough muscle tone, does not resist movement- floppy baby

A client at the clinic is considering the use of acupuncture to assist with smoking cessation. When describing this practice, the nurse would explain that which concept is involved as the focus?

Qi

Has been suggested to hold potential as an intervention to improve balance and reduce falls in older adults and has been used successfully to promote balance and coordination in other age groups

Qi Gong and Tai Chi

According to survey results, who are the most prevalent users of complementary or alternative therapies? a) Women, ages 25-40, with high school degree, who smoke b) Men, ages 50-65, with some high school, who smoke c) Men, ages 30-45, with college degree, who recently stopped smoking d) Women, ages 35-50, with college degree, former smokers

Women, ages 35-50, with college degree, former smokers Explanation: The most prevalent users of CAT are women, ages 30-69, with higher levels of education, who are former smokers.

Which of the following forms of medicine combines health promotion, establishment of a partnership with the client and practitioner, and captures an evolving model of health care? a. integrative health care and medicine b. homeopathic health care and medicine c. holistic health care and medicine d. palliative health care and medicine

a

The spouse of a client diagnosed with early memory loss asks about the use of ginkgo biloba as a dietary supplement. Which nursing assessment is essential prior to responding? a. Does the client have loss of recent memory or remote memory? b. Is the client taking warfarin or aspirin? c. Has the client ever had a myocardial infarction? d. What other dietary supplements does the client use?

b

Steppage

foot drop, loss of dorsiflexion with toes points and dragging along the floor causing a higher step than normal

The shaman or medicine man/woman possesses

the ability to access the spirit world

allopathic medicine

the term generally used to describe "traditional" medical care, dominant for about 100 years, which spearheaded remarkable advances in biotechnology, surgical interventions, pharmaceutical approaches, and diagnostic tools

holism

theory and philosophy that focuses on connections and interactions between parts of the whole

A nurse is assisting in recommending clients for chiropractic treatment. Which of the following clients should the nurse recommend?

A client who has shoulder pain

A client is asking for the nurse to explain acupuncture. What would the nurse tell the client?

Acupuncture is used to correct disharmony.

Enables organic beings and inorganic objects to communicate and interact •

Bioelectromagnetic

According to survey results, who are the most prevalent users of complementary or alternative therapies? • Men, ages 50-65, with some high school, who smoke • Men, ages 30-45, with college degree, who recently stopped smoking • Women, ages 25-40, with high school degree, who smoke Women, ages 35-50, with college degree, former smokers

Correct response: • Women, ages 35-50, with college degree, former smokers Explanation: The most prevalent users of CAT are women, ages 35-50, with higher levels of education, who are former smokers.

Adjunct therapy to occupational and physical therapy

Horticultural therapy

• Nurses assess for dietary supplement use and teach about benefits and risks and not considered medications

Naturopathy

3 types of Energy-Healing Therapies

Therapeutic touch • Healing touch • Sound healing

Aromatherapy is the use of

Therapeutic use of essential oils that is extracted from plants

A nurse is working as part of a team tasked with developing strategies to reduce the incidence of falls by adults in the facility. Based on the nurse's knowledge of complementary health approaches, which mind-body practice would the nurse suggest as a possible strategy? a. Tai chi b. Qi gong c. Yoga d. Guided imagery

a

A nurse cares for patients in a chiropractic office. What patient education might this nurse perform? Select all that apply. a)Applying heat or ice to an extremity b)Explaining the use of electrical stimulation c)Teaching a patient relaxation techniques d)Teaching a patient about a prescription e)Explaining an invasive procedure to a patient f)Teaching about dietary supplements

a, b, c, f. Chiropractors may combine the use of spinal adjustments and other manual therapies with several other treatments and approaches including heat and ice, electrical stimulation, relaxation techniques, rehabilitative and general exercise, counseling about weight and diet, and using dietary supplements. Chiropractors do not prescribe medication or perform invasive procedures.

When obtaining information for a database, which of the following represents a nurse commitment and interest in reflected integrative medicine (CAM)? a. "Which types of foods do you consume in 24 hours?" b. "Do you take any vitamins or minerals, and if so, what?" c. "Which prescription medications do you take daily?" d. "Which diseases do you suffer from and what are your allergies?"

b

The pregnant client with hyperemesis is not obtaining relief from medications. Which alternative therapy can the nurse advocate for the client to use? a. Homeopathy b. Qi Gong c. Acupuncture d. Shamanism

c. Acupuncture Acupuncture is used to minimize nausea and vomiting caused by pregnancy. It uses tiny needles placed at particular acupoints on the body to increase or decrease the flow of energy to contribute to healing. Shamanism is the use of a medicine man/woman to access the spirit world. Homeopathy is the use of substances for healing. Qi Gong is exercises or meditations that enhance systemic health. They are designed to restore the healing system, the body's innate intelligence, so it knows how to correct and heal itself.

______ is the use of hands on or near the body with the intent to heal or help

therapeutic touch

the nurse should support the client in choice of therapy as long as-

there is no potential for harm

Yin-yang therapy; Qi gong; and acupuncture are examples of what type of medicine?

traditional Chinese medicine

Homepathy is holistic is because

treats a person a whole rather than foucsing on illness

Which nursing action is an example of using integrative care? a) A nurse performing a home assessment of an older adult client recommends a referral to social services. b) A nurse monitors the heartbeat of a fetus and charts the height of the mother's fundus on the medical record. c) A nurse provides music therapy along with relaxation techniques for residents of a long-term facility. d) A nurse instructs a laboring woman to use breathing exercises, and assists with the administration of an epidural for her pain.

A nurse instructs a laboring woman to use breathing exercises, and assists with the administration of an epidural for her pain. Explanation: A nurse who uses integrative care uses some combination of allopathic medicine and CAM as seen with the nurse helping the woman in labor.

A nurse is assisting with teaching a class about U.S. Food and Drug Administration (FDA) regulation of nutritional supplements. Which of the following information should the nurse include in the teaching?

A nutritional supplement contains at least one nutritional component.

While assessing a client, the client tells the nurse that he is a follower of traditional Chinese medicine and the concept of qi. Based on the nurse's understanding of this concept, which treatment modality would the nurse expect the client to mention? A) Acupuncture B) Allopathy C) Physiotherapy D) Therapeutic touch

A) Acupuncture

Healing environments

Created by providing knowledge, skills, and support

What is the allopathic medicine emphasis?

Emphasis on disease and high technology such as drugs, surgery and radiation

Which of the following has been known to achieve benefits for clients with cancer through the use of the mind to visualize a positive physiologic effect? a) Humor b) Biofeedback c) Imagery d) Hypnosis

Imagery Explanation: Imagery is a physiologic technique that uses the mind to visualize a positive physiologic effect. When using imagery, clients conjure up mental images of their body waging and winning a battle with the disease process.

Includes drive to become all one can Bound to intuition, creativity, motivation Relationship with oneself, others, and a higher power

Spirituality

Examples of healing and treatment in Native American Healing

Sweat lodge, singing, dancing, and vision quest (includes fasting and solitude) • Smudging, drumming, chanting, healing touch, acupuncture, herbs

Allopathic medicine is less effective in the care of clients with: a. Chronic illnesses. b. Acute illnesses. c. Emergency situations. d. Surgical needs.

a

Which of the following is a principle of Therapeutic Touch (TT)? a. Human beings have natural abilities to transform and transcend their conditions of living. b. Human beings are a closed energy system. c. Human beings are unilaterally symmetrical. d. Illness is a balance in an individual's energy field.

a

Ayurveda

a science of life that delineates the diet, medicines, and behaviors that are beneficial or harmful for life and considers that balance among people, the environment, and the larger cosmos is integral to human health

although sometimes ssumed that all homeopathic redeies are highly diluted and therefore unlikely to cause harm, some products labeled as homeopathic contain substanial amounts of:

active ingredients, thus could cause side effects and drug interactions. **little evidence supports homeopathy as an effective treatment

either increases or decreases the flow of qu along the meridian, restoring the balance of yin and yang -- change in flow contiributes to healing

acupuncture

Kyphosis (hunchback)

overtime hunchbacking, with thoracic and sacral curve towards back of the body - senile kyphosis where gelatin cartilage harden and will cause compressing.

paraplegia

paralysis from the waist down legs

A nurse is assisting with teaching a client, who has a history of alcohol use disorder and is to be discharged following cataract surgery, about non-pharmacological therapies. Which of the following client statements indicates an understanding of the teaching?

"I will practice meditating when I feel stressed."

The nurse in a medical unit is collecting a client's history and asks the client about the use of complementary and alternative therapies. The client asks why the nurse needs to know about this. What is the nurse's best response?

"It's important that we list all of your complementary health practices used to provide a full picture of what you do to manage your health."

A nurse is reinforcing teaching about traditional Chinese medicine (TCM) to a client. Which of the following statements by the client demonstrates an understanding of the teaching?

"I will practice Tai Chi every day."

Goal of care of healing touch

Accelerate person's own healing process and • Facilitate healing of body, mind, emotions, spirit

Which nursing action exemplifies the use of integrative care? a. A nurse instructs a laboring woman to use breathing exercises, and assists with the administration of an epidural for her pain. b. A nurse monitors the heartbeat of a fetus and charts the height of the mother's fundus on the medical record. c. A nurse provides music therapy along with relaxation techniques for residents of a long-term facility. d. A nurse performing a home assessment of an older adult client recommends a referral to social services.

a

______ not included in the conventional scope of practice

alternative therapy

A home care nurse delivers care that incorporates a philosophy that focuses on connections and interactions between parts of the whole. Which term best describes this philosophy? a. Homeopathy b. Holism c. Complementary therapy d. Integrative care

b. Holism Holism is a theory and philosophy that focuses on connections and interactions between parts of the whole. Complementary therapy can be used to complement traditional medical interventions. Integrative care often incorporates sharing the responsibility in coordinating the best possible treatment plan for a client, including the client's choices for care and the provider's expertise. Homeopathy is based on the belief of supporting the body while the symptoms are allowed to "run their course."

Ashtanga Yoga

focuses on synchronizing breath with a fast-paced series of postures

therapeutic touch (TT)

an alternative therapy that involves using one's hands to consciously direct an energy exchange from the practitioner to the patient to facilitate healing or pain relief

A client who is receiving chemotherapy and experiencing significant nausea asks the nurse about using aromatherapy to help alleviate the nausea. Which essential oil would the nurse most likely suggest to address the client's nausea? a. Lavender b. Chamomile c. Cedarwood d. Ginger

d. Ginger Ginger or peppermint is an appropriate essential oil choice for addressing nausea. Chamomile, lavender, and cedarwood are used for insomnia.

CAT: involves the use of energy fields such as magnetic or biofields (energy fields that some believe surround and penetrate the human body)

energy medicine

_____ involves using the 5 senses to imagine an event or body process unfolding according to a plan.

imagery

Myoclonus

rhythmic asculatating, stretch reflex usually caused by leisons of upper motor neuron and hyperreflexia

The nurse in a medical unit is collecting a client's history and asks the client about the use of complementary and alternative therapies. The client asks why the nurse needs to know about this. What is the nurse's best response? a) "I am just curious on what types of treatments are used by people." b) "I want to make sure you understand all the risks of these treatments." c) "It's important that we list all your home care needs for the hospital." d) "It will help me so that I can recommend use of these for other clients."

"It's important that we list all your home care needs for the hospital." Explanation: Nurses need to include complementary and alternative therapy in assessment of the client's current treatments to ensure an understanding of the safety and effectiveness of the treatments, particularly if the client is also receiving allopathic treatment. The nurse would want to be positive with this questioning so that the client is not defensive regarding any complementary and alternative treatments.

During a client interview, the client tells the nurse about using ginkgo biloba to improve memory. When reviewing the client's medication history, which medication if used by the client would be a cause for concern? • Warfarin • Acetaminophen • Azithromycin • Guaifenesin

Correct response: • Warfarin Explanation: Ginkgo biloba, the most widely sold herb in Europe and used by many to improve memory, affects platelet function and thus should not be used concurrently with warfarin or aspirin. It appears that acetaminophen, guaifenesin, and azithromycin would not be a concern.

What is alternative therapy?

Refers to one that is utilized instead of conventional treatment

There is (blank) in integrative health?

There is HIGH QUALITY EVIDENCE OF SAFETY AND EFFECTIVENESS

In which CAM modality are the concepts of Yin and Yang important for diagnosis? a) Traditional Chinese Medicine b) Therapeutic Touch (TT) c) meditation d) botanicals

Traditional Chinese Medicine Explanation: Diagnosis in Traditional Chinese Medicine is based in the balance, or lack thereof, of Yin and Yang. Yin and Yang are both aspects of qi, which is considered a vital life force.

Nurses should inquire about healing modalities client may have used previously

Healing modalities and you want a good body of knowleged

The nursing instructor is discussing alternative therapy with a group of students. She explains that living organisms are "continuously connecting and interacting with their environment." Furthermore, the connecting and interacting signifies that the human body is a unified dynamic whole. The instructor is describing what theory to the students?

Holism perspective

A nurse is caring for a client who asks about using aromatherapy for nausea. Which of the following scents should the nurse recommend? (Select all that apply.)

Peppermint Ginger Orange

Gives people meaning and purpose in lives

Spirituality

A nurse is caring for a patient who has crippling rheumatoid arthritis. Which nursing intervention best represents the use of integrative care? a) The nurse administers naproxen and uses guided imagery to take the patient's mind off the pain. b) The nurse prepares the patient's health care provider-approved herbal tea and uses meditation to relax the patient prior to bed. c) The nurse administers naproxen and performs prescribed range-of-motion exercises. d) The nurse arranges for acupuncture for the patient and designs a menu high in omega-3 fatty acids.

a. Adding guided imagery (CHA) to the administration of pain medications (allopathy) is an example of integrative care. A person who uses integrative care uses some combination of allopathic medicine and CHA.

A nurse works for a health care provider who practices the naturopathic system of medicine. What is the focus of nursing actions based on this type of medical practice? Select all that apply. a) Treating the symptoms of the disease b) Providing patient education c) Focusing on treating individual body systems d) Making appropriate interventions to prevent illness e) Believing in the healing power of nature f) Encouraging patients to take responsibility for their own health

b, d, e, f. Naturopathic medicine is not only a system of medicine, but also a way of life, with emphasis on patient responsibility, patient education, health maintenance, and disease prevention. Its principles include minimizing harmful side effects and avoiding suppression of symptoms, educating patients and encouraging them to take responsibility for their own health, treating the whole person, preventing illness, believing in the healing power of nature, and treating the cause of a disease or condition rather than its symptoms.

what is ginko biloba frequently used for?

dilation of cerebral blood vessels and reduce symptoms of memory loss and mental confusion. interferes with platelet function: do not take with warfarin or aspirin - increased bleeding

focuses on evoking pleasant images to replace negative or stressful feelings and to promote relaxation

guided imagery

what is acupuncture used for?

reducing pain, promoting adherence to substance abuse programs, and minimizing nausea and vomiting due to chemotherapy and pregnancy

Chiropractic health care

Correct spinal dysfunction

A nurse is caring for a client who asks about naturopathy. The nurse should include that which of the following is a component of naturopathy? (Select all that apply.)

Dietary supplements Exercise Traditional Chinese medicine Homeopathy

A nurse is caring for a hospitalized client who states: "I feel so sick all the time; my aura must be disturbed by all of these bad force fields." What is an appropriate NANDA-I diagnosis for this client?

Disturbed energy field

Stimulates five senses • Provides leisure activities • Improves motor function • Provides sense of achievement • Improves self-esteem

Horticultural Therapy

Resident animals

Live at long-term health care facilities and Gravitate to most isolated or depressed clients

According to survey results, who are the most prevalent users of complementary or alternative therapies? a. Women, ages 35-50, with college degree, former smokers b. Men, ages 50-65, with some high school, who smoke c. Women, ages 25-40, with high school degree, who smoke d. Men, ages 30-45, with college degree, who recently stopped smoking

a

The nurse is caring for a client whose treatment has been based on the Ayurveda medical system. Which nursing intervention incorporates this client's beliefs into the nursing plan? a. Including the client's shaman in the plan of care b. Basing practice on the yin-yang theory c. Helping the client to balance his or her dosha d. Preparing the client for exercises that help him or her regulate qi

c. Helping the client to balance his or her dosha Central to Ayurvedic medicine is an understanding of th client's basic constitution, or dosha. The three doshas in Ayurvedic medicine are vata (changeable), pitta (intense), and kapha (relaxed). Yin-yang and qi are associated with traditional Chinese medicine. A shaman is associated with shamanism (involves the spirit world).

Harmony between what in native american healing

mind and body

On the advice of friends, a client on a palliative care unit has requested acupuncture. What it is the goal of this form of CAM? a) allowing accumulated toxins to be released from the body b) altering the client's perception and acceptance of reality c) restoring a healthy flow of energy along the meridians of the body d) reconnecting the client's body, spirit, and emotions

restoring a healthy flow of energy along the meridians of the body Explanation: Acupuncture either increases or decreases the flow of qi along the meridians of the body. It does not directly address the relationships between body, spirit, and emotions, nor does it aim to release toxins or change an individual's perception of reality.

used to treat enlarged prostate; has no effects on bleeding

saw palmetto

how can nurses assist with imagery?

see if recorded imagery material is available on the nursing unit or assist pt with writing own script and record it use "outcome imagery" -- might consist of using a picture of photgraph to visualize the desired outcome in a body part or in a situation. pts can add to the visual cue and develop a total image during a painful stressful event (i.e IV line being started) the pt can "go to a favorite place" and imagine being there with all the pleasant experiences related to that space

A nurse is applying healing touch to a postoperative client's wound. What benefit of healing touch should the nurse emphasize to the client's family? A) It will decrease the need for fluids after surgery. B) It will enhance family relationships. C) It will help the wound heal. D) It will improve the nurse-client relationship.

C) It will help the wound heal

Herbals in Ayurveda

For preventive or regenerative purposes • Treatment for specific disorders

A nurse is caring for a client who follows a vegan diet and reports lethargy. Which of the following interventions should the nurse implement?

Request a prescription for vitamin B12 supplements.

what can herbs be used for?

herbs can be used for treatment fo disease or reduction of symptoms

qi gong

system of posture, exercise (both gentle and dynamic), breathing techniques, and visualization that regulates the qi

Which of these assessment findings indicate a positive outcome for a client after acupuncture? A) The client reports a reduction in pain level to 3 out of 10. B) The client has improved balance and coordination. C) The client has lost 8 lb (3.63 kg) over the last 2 months. D) The client has increased his or her flexibility and strength.

A) The client reports a reduction in pain level to 3 out of 10

Cultural traditions and healing practices are part of medical mainstream in other cultures.

Healing modalities

A client is extremely anxious while awaiting the results of a biopsy. What action by the nurse will assist the client with progressive relaxation techniques? a. Have the client tighten and release different muscle groups. b. Have the client talk about the procedure c. Have the client walk around the waiting room d. Apply pressure to the temple area to increase the flow of energy.

a

A nurse is providing a lecture on CHAs to a group of patients in a rehabilitation facility. Which teaching point should the nurse include? a) CHAs are safe interventions used to supplement traditional care. b) Many patients use CHA as outpatients but do not wish to continue as inpatients. c) Many nurses are expanding their clinical practice by incorporating CHA to meet the demands of patients. d) Most complementary and alternative therapies are relatively new and their efficacy has not been established.

c. Many nurses are expanding their clinical practice by incorporating CHA. Although CHA may seem totally safe, some therapies have led to harmful and, at times, lethal outcomes. Many patients use these types of therapies as outpatients and want to continue their use as inpatients. Although the use of most complementary and alternative therapies predates modern medicine, it was not until recently that nursing and medical schools began to teach about their use.

A nurse is teaching an overweight patient the holistic approach to choosing foods. Which teaching points would the nurse include? (Select all that apply.) a) Reduce intake of refined and natural sugars. b) Consider adopting a vegetarian diet. c) Eat foods that are in season. d) Avoid organically grown foods. e) Replace refined sugars with artificial sweeteners. f) Increase intake of dairy products

• Eat foods that are in season. • Reduce intake of refined and natural sugars. • Consider adopting a vegetarian diet. Explanation: Eating foods that are in season is suggested because foods produced "out of season" are treated with chemicals to ripen them since they are picked before ripening. Reducing the intake of refined and natural sugars is best because of the negative effects of these substances. A vegetarian diet can be beneficial because animals are increasingly fed antibiotics and potentially contaminated foods. Also, increasingly, fish are being contaminated with mercury and other pollutants.

A nurse who has incorporated complementary and alternative medicine (CAM) into nursing practice is caring for a client in a short-term care facility. Which examples of nursing interventions are based on CAM? Select all that apply. a) The nurse encourages the client to join a yoga class. b) The nurse administers pain medication prescribed by the primary care provider. c) The nurse teaches the client how to meditate. d) The nurse investigates herbs that may stimulate the client's immune system. e) The nurse uses guided imagery to relieve client anxiety. f) The nurse schedules diagnostic tests for the client.

• The nurse encourages the client to join a yoga class. • The nurse teaches the client how to meditate. • The nurse investigates herbs that may stimulate the client's immune system. • The nurse uses guided imagery to relieve client anxiety. Explanation: The term complementary and alternative medicine refers to interventions that are complementary therapies (CAM—they can be used with traditional medical interventions and thus complement them) as well as alternative modalities (not included in the scope of conventional medical care). Herbal remedies, yoga, meditation, and guided imagery are all forms of CAM. Administering pain medication and scheduling diagnostic tests are traditional allopathic (biomedicine) therapies

A nurse manager who works in a hospital setting is researching the use of energy healing to use as an integrative care practice. Which patient would be the best candidate for this type of CHA? a) A patient who is anxious about residual pain from cervical spinal surgery b) A patient who is experiencing abdominal discomfort c) A patient who has chronic pain from diabetes d) A patient who has frequent cluster headaches

a. Energy healing is focused on pain that lingers after an injury heals, as well as pain complicated by trauma, anxiety, or depression. Nutritional and herbal remedies treat all chronic pain, but especially abdominal discomfort, headaches, and inflammatory conditions, such as rheumatoid arthritis.

involves using all 5 senses to imagine an event or body process unfolding according to a plan

guided imagery

two unconvential theorys of homeopathy:

1. "like cures like" -- the notion that adisease can be cured by a substance that produces similar symptoms in helathy people 2. "law of minimum dose" -- the notion that the lower the dose of meds, the greater its effectiveness. many homeopathic remedies are so diluted that no molecules of the original substance remain.

A nurse is teaching a client with anxiety about the use of meditation. Which of these are important features of meditation? Select all that apply.

A quiet location An open attitude A comfortable position

What is conventional medical interventions?

A system in which medical doctors and other healthcare professionals (such as nurses, pharmacists, and therapists) treat symptoms and diseases using drugs, radiation, or surgery

A nurse is caring for a hospitalized patient who states: "I feel so sick all the time, my aura must be disturbed by all of these bad force fields." What is an appropriate NANDA diagnosis for this patient? a) Impaired coping b) Hopelessness c) Social isolation d) Disturbed energy field

Disturbed energy field Explanation: Defining characteristics of the disturbed energy field relate to movement, sounds, temperature change, visual changes, and disruption. The aura the patient is referring to in this scenario is a vague, luminous glow surrounding something that may be an information containing electromagnetic field, which can be likened to the data contained within a computer.

Which is the most important role of the nurse in using complementary and alternative therapies? a. Practicing guided imagery prior to surgery b. Administering herbal supplements for anxiety c. Educating the public about safety and effectiveness d. Providing nutrition supplements for weight gain

c. Educating the public about safety and effectiveness The most important role of the nurse in using CAT is educating the public about the safety and effectiveness of using the therapies. Administering herbal supplements for anxiety and providing nutrition supplements for weight gain may be considered part of a treatment plan but they are not the most important aspects of CAT. Guided imagery is one aspect of treatment included in CAT.

_____ therapy is one of the greatest risk factors for adverse effects when combined with other conventional drugs

herbal therapy

A nurse is assisting with an in-service for staff members about culturally competent care. Which of the following information should the nurse include about traditional Native American beliefs?

Music can produce healing energy.

A holistic nurse is caring for a client experiencing anxiety. The nurse assesses the client's energy field then uses her hands to promote comfort and a feeling of well being. This practice is termed: a) massage therapy. b) hatha yoga. c) aromatherapy. d) Therapeutic Touch (TT).

Therapeutic Touch (TT). Explanation: Therapeutic touch involves "unruffling," or clearing, congested areas of energy in the body and redirecting this energy. After assessing a client's "energy field," the nurse uses therapeutic touch to promote comfort, relaxation, healing, and a sense of well-being. Massage therapy is performed by a licensed massage therapist. Hatha yoga is the practice of slow and balanced postures taught by a yoga instructor. Aromatherapy involves the use of scents, not touch, as therapy.

A client is extremely anxious while awaiting the results of a biopsy. What action by the nurse will assist the client with progressive relaxation techniques? a. Have the client tighten and release different muscle groups. b. Apply pressure to the temple area to increase the flow of energy. c. Have the client walk around the waiting room d. Have the client talk about the procedure

a. Have the client tighten and release different muscle groups. Relaxation is to progressively tighten and release different muscle groups to relax. This is called progressive relaxation. To visualize is the use of guided imagery to mentally challenge one's physical reality. To focus the mind is meditation. To apply pressure is using acupressure to balance and increase flow of energy.

The nurse is teaching a class on methods of relaxation to be used for stress management. Which methods should the nurse teach the participants? Select all that apply. a. Biofeedback b. Acupuncture c. Focused breathing d. Imagery e. Meditation

e. Meditation a. Biofeedback d. Imagery c. Focused breathing Methods of relaxation that can be used for stress management are: meditation, biofeedback, imagery, and focused breathing. Acupuncture is used for pain relief, minimizing nausea and vomiting, and substance use recovery adherence.

how does the FDA in the US classify herbs?

food, not a drug (important because herbs are not regulated for quality and potency as drugs are and herbal formulas can be sold without being studied to see if they are helpful or harmful)

a theory that focuses on connections and interactions between parts of the whole

holism

A nurse is explaining the practice of Ayurveda to a newly licensed nurse. The nurse should include that Ayurveda is based on establishing a balance between the body, mind, and spirit with which of the following elements? (Select all that apply.)

Earth Sky Water

Which question or statement to a client conveys acceptance? a. "What helpful herbal supplements are you taking?" b. "Have you ever discussed taking vitamins and supplements with your doctor?" c. "Will you please share with me the prescription medicines and vitamins you take?" d. "You know supplements can be harmful. Do you take any supplements?"

c. "Will you please share with me the prescription medicines and vitamins you take?" Conveying an open and nonjudgmental attitude is a key to accurate collection of this information. This begins with the initial contact with a client and includes a basic assessment of the client's use of CAM. A basic question asking to share information about prescription and supplements conveys acceptance. If the nurse ask about discussion with a health care provider, the client may infer either approval or disapproval of supplement use. If the nurse states that supplements can be harmful, the client may infer disapproval of their use. Conversely,if the nurse states that supplements are helpful, the client may infer approval.

The mind and body are connected in the provision of care. This statement describes:

holistic care.

The mind and body are connected in the provision of care. This statement describes: a) holistic care. b) altruistic care. c) homeopathic care. d) allopathic care.

holistic care. Explanation: Holistic interventions focus on the interrelated needs of body, mind, emotions, and spirit.

believe that when symptoms are suppressed by medication, the condition "goes deeper" into the body, making it ultimately more difficult to cure.

homeopathics

Paresis

muscle weakness or imparied strength

Sarah Stevens is an 85-year-old woman who is in the hospital recovering from hip replacement surgery. She is scheduled to go home in 2 days. According to the Health Belief Model, which factors would be important to assess before Sarah goes home? Select all that apply. a) what she has heard about recovering from this procedure b) her expectations for recovery c) her living environment d) how she has recovered from previous procedures e) whether or not she feels safe in her home

• her expectations for recovery • what she has heard about recovering from this procedure • how she has recovered from previous procedures • her living environment • whether or not she feels safe in her home Explanation: The Health Belief Model asserts that certain factors influence a person's health and the way they respond to illness. These factors include personal expectations in relation to health and illness, earlier experiences with health and illness, sociocultural context, and age and developmental state.

While assessing a client, the client tells the nurse that he is a follower of traditional Chinese medicine and the concept of qi. Based on the nurse's understanding of this concept, which treatment modality would the nurse expect the client to mention? a) Allopathy b) Therapeutic Touch (TT) c) Acupuncture d) Physiotherapy

Acupuncture Explanation: Acupuncture is based on energy regulation of qi through meridians. It is a core principle in traditional Chinese medicine. Therapeutic Touch (TT) is not a form of traditional Chinese medicine and is based on the consciously directed process of energy exchange. Physiotherapy and allopathy are not based on the concept of qi.

A client is asking for the nurse to explain acupuncture. What would the nurse tell the client? a) Acupuncture is used to correct disharmony. b) Acupuncture is beneficial to creating a mood of distraction. c) Acupuncture is a dangerous option for the treatment of disease. d) Acupuncture is only done in Eastern countries.

Acupuncture is used to correct disharmony. Explanation: Acupuncture can be used to correct disharmony or prevent disharmony from developing.

The client who is newly diagnosed with diabetes mellitus type 2 is concerned about eating products with sugar in them. What information does the nurse explain to the client regarding the use of sugar? a) It assists with cellular absorption of nutrients. b) An excess increases demand on the pancreas. c) Healthy amounts are usually consumed. d) It increases natural bacterial flora in the GI tract.

An excess increases demand on the pancreas. Explanation: Excess sugar increases demand on the pancreas; therefore, less sugar should be used. The client who has diabetes mellitus type 2 already has a compromised pancreas. Further stress to the pancreas can cause further injury to the organ.

A nurse can best help a client who is undergoing chemotherapy and using guided imagery with this by doing which of the following? a) Assisting the client to find an appropriate imagery tape to use. b) Explaining to the client that it is not a good idea to record their own imagery tape. c) Promoting the client's use of imagery only after a stressful event occurs. d) Helping the client learn about the different poses that can be performed.

Assisting the client to find an appropriate imagery tape to use. Explanation: Clients use imagery to help with relaxation and this can use all five senses. Many times, clients use a guided imagery tape that includes a script to help with imagery; clients also can record their own tape.

A nurse can best help a client who is undergoing chemotherapy and using guided imagery with this by doing which of the following? A) Helping the client learn about the different poses that can be performed. B) Assisting the client to find an appropriate imagery tape to use. C) Promoting the client's use of imagery only after a stressful event occurs. D) Explaining to the client that it is not a good idea to record their own imagery tape.

B) Assisting the client to find an appropriate imagery tape to use.

Which of the following would be the least consistent with the Native American medicine view of disease? a) Balance of yin and yang b) A result of negative thinking c) Violation of a taboo d) Disharmony with Mother Earth

Balance of yin and yang Explanation: The Native Americans view disease as a disharmony with Mother Earth, a result of negative thinking, or violation of a taboo. Chinese medicine proposes that health is the outcome of balancing yin and yang.

The nurse is educating a client on how to use herbs and supplements as part of an integrated treatment plan. Which teaching points would the nurse include? Select all that apply. A )Use the Internet to buy herbs and supplements. B) Whenever possible, buy products with more than one ingredient. C) Buy herbs and supplements that are standardized. D) Give the product adequate time to work. E) Be knowledgeable about the product and its therapeutic actions. F) Take a higher than recommended dose of herbs to initiate the therapeutic effect.

C, D, E

The pregnant client with hyperemesis is not obtaining relief from medications. Which alternative therapy can the nurse advocate for the client to use? • Shamanism • Qi Gong • Homeopathy • Acupuncture

Correct response: • Acupuncture Explanation: Acupuncture is used to minimize nausea and vomiting caused by pregnancy. It uses tiny needles placed at particular acupoints on the body to increase or decrease the flow of energy to contribute to healing. Shamanism is the use of a medicine man/woman to access the spirit world. Homeopathy is the use of substances for healing. Qi Gong is exercises or meditations that enhance systemic health. They are designed to restore the healing system, the body's innate intelligence, so it knows how to correct and heal itself.

A nurse can best help a client who is undergoing chemotherapy and using guided imagery with this by doing which of the following? • Promoting the client's use of imagery only after a stressful event occurs. • Explaining to the client that it is not a good idea to record their own imagery tape. • Assisting the client to find an appropriate imagery tape to use. • Helping the client learn about the different poses that can be performed.

Correct response: • Assisting the client to find an appropriate imagery tape to use. Explanation: Clients use imagery to help with relaxation and this can use all five senses. Many times, clients use a guided imagery tape that includes a script to help with imagery; clients also can record their own tape. Clients do not necessarily use poses with this; this would be more related to use of yoga. Guided imagery can help both during and after a stressful event to help the client relax.

Which is the most important role of the nurse in using complementary and alternative therapies? • Educating the public about safety and effectiveness • Providing nutrition supplements for weight gain • Administering herbal supplements for anxiety • Practicing guided imagery prior to surgery

Correct response: • Educating the public about safety and effectiveness Explanation: The most important role of the nurse in using CAT is educating the public about the safety and effectiveness of using the therapies. Administering herbal supplements for anxiety and providing nutrition supplements for weight gain may be considered part of a treatment plan but they are not the most important aspects of CAT. Guided imagery is one aspect of treatment included in CAT.

The client who is newly diagnosed with diabetes mellitus type 2 is concerned about eating products with sugar in them. What information does the nurse explain to the client regarding the use of sugar? • Sugar assists with cellular absorption of nutrients. • Excess sugar increases demand on the pancreas. • Healthy amounts of sugar are usually consumed. • Sugar increases natural bacterial flora in the GI tract.

Correct response: • Excess sugar increases demand on the pancreas. Explanation: Excess sugar increases demand on the pancreas; therefore, less sugar should be consumed. The client who has diabetes mellitus type 2 may have a compromised pancreas. Further stress to the pancreas can cause further injury to the organ.

A nurse is teaching a client experiencing stress about how relaxation helps to reduce the effects of stress on the body. Which underlying concept would the nurse integrate into the explanation about how relaxation works? • Activates natural pleasure centers • Slows circulation throughout the body • Helps to increase the effects of parasympathetic nervous system on the mind and body • Increases the body's natural immunity

Correct response: • Helps to increase the effects of parasympathetic nervous system on the mind and body Explanation: Relaxation techniques promote parasympathetic nervous system activity, helping to reduce sympathetic activity and restore the balance of the two systems. The ultimate goal is to increase the parasympathetic system influence in the mind-body and thus reduce the effect of stress and stress-related illness on the body. Natural products can boost the body's immunity. Energy healing activates natural pleasure centers. Hands-on techniques such as massage promote circulation throughout the body.

A home care nurse delivers care that incorporates a philosophy that focuses on connections and interactions between parts of the whole. Which term best describes this philosophy? • Complementary therapy • Integrative care • Homeopathy • Holism

Correct response: • Holism Explanation: Holism is a theory and philosophy that focuses on connections and interactions between parts of the whole. Complementary therapy can be used to complement traditional medical interventions. Integrative care often incorporates sharing the responsibility in coordinating the best possible treatment plan for a client, including the client's choices for care and the provider's expertise. Homeopathy is based on the belief of supporting the body while the symptoms are allowed to "run their course."

The nursing instructor is discussing alternative therapy with a group of students. She explains that living organisms are "continuously connecting and interacting with their environment." Furthermore, the connecting and interacting signifies that the human body is a unified dynamic whole. The instructor is describing what theory to the students? • Holism perspective • Integrative perspective • Allopathic perspective • Medical system perspective

Correct response: • Holism perspective Explanation: The instructor is describing the definition of holism. Integrative perspective is combining allopathic and complementary and alternative therapy (CAM). The medical system perspective is based on a philosophy and theory about health and illness along with specific types of treatment that may be based in culture. The allopathic perspective is based in biomedicine. Holism is the only perspective that is looking at parts interacting with each other and the environment

Which principle does not encompass the basic goals of integrative medicine? • Establish a partnership between client and practitioner. • Focus on promoting health and preventing illness, as well as treating disease. • Facilitate the body's innate healing abilities. Reject allopathic medicine and embrace CAM practices.

Correct response: • Reject allopathic medicine and embrace CAM practices. Explanation: Integrative medicine is healing-oriented medicine that takes account of the whole person (body, mind, spirit, and community), including all aspects of lifestyle. It does not reject allopathic medicine and CAM practices. It includes establishing a partnership between client and practitioner, facilitating the body's innate healing abilities, and focusing on promoting health and preventing illness, as well as treating disease.

Which is the most important role of the nurse in using complementary and alternative therapies? a) Practicing guided imagery prior to surgery b) Providing nutrition supplements for weight gain c) Administering herbal supplements for anxiety d) Educating the public about safety and effectiveness

Educating the public about safety and effectiveness\ Explanation: The most important role of the nurse in using CAT is educating the public about the safety and effectiveness of using the therapies. Herbal supplements for anxiety and nutrition supplements may be considered part of a treatment plan but they are not the most important aspects of CAT. Guided imagery is one aspect of treatment included in CAT.

A nursing instructor invites a nurse who specializes in healing touch to her classroom to discuss with students the effect healing touch has on pain management. This is an example of what? a) Allowing for discussion about adjuvant therapy b) Familiarizing students with CAM modalities c) Instructing students so they can use healing touch d) Evaluating the effect of healing touch on her students

Familiarizing students with CAM modalities Explanation: Some nurses may choose to become trained in particular CAM modalities and then integrate them into their own practice.

A nurse is teaching a class on holistic nutritional therapy to a group of young and middle-aged adults in the community. The nurse creates a menu of food and beverage choices and asks the group the select the best choices. The nurse determines that the teaching was successful when the group chooses which food? Select all that apply.

Fresh corn on the cob in season Organically grown chicken Bottled spring water

A nurse is completing a preoperative health history for a client who reports taking several herbal supplements. Which of the following herbal supplements is the priority for the nurse to report?

Ginkgo biloba

A nurse is providing care to a client who has come to the outpatient clinic for chemotherapy. The client tells the nurse that to cope with the stress of chemotherapy, he uses a technique in which he "goes to my happy place, the beach, and I picture myself lying there under the warm sun, with the sound of the waves lapping at the shore." The nurse interprets this as which technique?

Guided imagery

Imagery refers to two -way communication between conscious and unconscious mind and involves whole body and senses • State of focused attention that encourages changes in attitude, behavior, and physiological reactions • Helps stop troublesome thoughts

Guided imagery

Healing touch

Hand-mediated biofield therapies • Includes Therapeutic Touch (TT), Healing Touch (HT), and Reiki • Use of hands on or near with intention to heal

Synthesis of medical-curing and nursing-healing approach • Created when time is taken to be with clients in deeply caring ways

Healing environment

A client asks the nurse about the use of healing touch. Which statement regarding healing touch is accurate? a) Healing touch does not use injections. b) Healing touch is too expensive for most clients. c) Supplemental music is used during healing touch. d) Aromatherapy is used in healing touch.

Healing touch does not use injections. Explanation: Healing touch has been shown to be effective in helping clients relax and improve the healing process. It is not expensive because it involves no special equipment, including insertion of tubes or injections. Aromatherapy and supplemental music are not routinely used in healing touch

The nursing instructor is discussing alternative therapy with a group of students. She explains that living organisms are "continuously connecting and interacting with their environment." Furthermore, the connecting and interacting signifies that the human body is a unified dynamic whole. The instructor is describing what theory to the students? a) Integrative perspective b) Allopathic perspective c) Medical system perspective d) Holism perspective

Holism perspective Explanation: The instructor is describing the definition of holism. Integrative perspective is combining allopathic and complementary and alternative therapy (CAM). The medical system perspective is based on a philosophy and theory about health and illness along with specific types of treatment that may be based in culture. The allopathic perspective is based in biomedicine. Holism is the only perspective that is looking at parts interacting with each other and the environment.

A nurse is planning complementary therapy for a client who has breast cancer. Which of the following treatments is a complementary therapy that the nurse can implement?

Massage therapy

How are aromatherapies used?

Massaged into skin, inhaled, placed in baths, used as compresses, or mixed into ointments in order to calm, stimulate and improve sleep and change eating habits and boost immunity

Relaxation • Meditation • Guided imagery • Yoga • Qi Gong and Tai Chi • Acupuncture • Chiropractic health care • Aromatherapy

Mind-Body Practices

Herbal medicine

Most herbal medicines present no danger if taken appropriately. • Some can cause serious side effects if taken in excess or over a prolonged period.

The client has been diagnosed with a disease and is seeking information about naturopathy. The nurse, explaining about naturopathy, encourages the client to do what?

Obtain adequate sleep each night.

An occupational therapist conducts daily relaxation exercises with clients who are receiving treatment on the psychiatric unit of a hospital. Stimulation of which of the following components of the nervous system will result in relaxation for the clients?

Parasympathetic nervous system

An occupational therapist conducts daily relaxation exercises with clients who are receiving treatment on the psychiatric unit of a hospital. Stimulation of which of the following components of the nervous system will result in relaxation for the clients? a) Parasympathetic nervous system b) Autonomic nervous system c) Sympathetic nervous system d) Central nervous system

Parasympathetic nervous system Explanation: Stimulation of the parasympathetic nervous system results in an alleviation of stress. Direct stimulation of the sympathetic nervous system increases stress. The autonomic nervous system encompasses both the sympathetic and parasympathetic divisions. The central nervous system consists of the brain and spinal cord; neither is stimulated to facilitate relaxation.

Having a holistic health philosophy or a transformational experience that changes one's worldview and wanting greater control over one's own health

Reason of use of complementary health

How does chiropractic health care correct spinal dysfunction?

Reestablish shock absorption, leverage, and range of motion • Muscles and ligaments strengthened • Spinal rehabilitative exercises

Which principle does not encompass the basic goals of integrative medicine? a) Facilitate the body's innate healing abilities. b) Focus on promoting health and preventing illness, as well as treating disease. c) Establish a partnership between client and practitioner. d) Reject allopathic medicine and embrace CAM practices.

Reject allopathic medicine and embrace CAM practices. Explanation: Integrative medicine is healing-oriented medicine that takes account of the whole person (body, mind, spirit, and community), including all aspects of lifestyle. It does not reject allopathic medicine and CAM practices. It includes establishing a partnership between client and practitioner, facilitating the body's innate healing abilities, and focusing on promoting health and preventing illness, as well as treating disease.

Why would one use complementary health?

Seeking ways to improve health and well-being or relieve symptoms associated with chronic, even terminal illnesses or the side effects of conventional treatments for them

Which of these assessment findings indicate a positive outcome for a client after acupuncture? a) The client has increased his or her flexibility and strength. b) The client has lost 8 pounds over the last 2 months. c) The client reports a reduction in pain level to 3 out of 10. d) The client has improved balance and coordination.

The client reports a reduction in pain level to 3 out of 10. Explanation: Acupuncture can help with reduction of pain and nausea with clients. It is not a treatment for balance and coordination, strength and flexibility, or weight loss.

A nurse who has incorporated complementary and alternative medicine (CAM) into nursing practice is caring for a client in a short-term care facility. Which examples of nursing interventions are based on CAM? Select all that apply.

The nurse investigates herbs that may stimulate the client's immune system. The nurse encourages the client to join a yoga class. The nurse teaches the client how to meditate. The nurse uses guided imagery to relieve client anxiety.

A nurse who "unblocks" and "clears" congested areas of energy in a client's body to promote comfort is applying the phenomenon known as: a) "Unruffling" touch b) Tactile manipulation c) Interpersonal touch d) Therapeutic Touch (TT)

Therapeutic Touch (TT) Explanation: Therapeutic Touch (TT) involves "unruffling," or clearing, congested areas of energy in the body and redirecting this energy. After assessing a client's "energy field," the nurse uses therapeutic touch to promote comfort, relaxation, healing, and a sense of well-being

A nurse is assisting with teaching a newly licensed nurse about holistic nursing. Which of the following information should the nurse include? (Select all that apply.)

Use deep breathing to promote relaxation in clients. Encourage clients to use guided imagery to decrease pain. Apply essential oils to treat anxiety in clients.

A nurse is caring for a client admitted for a prolonged stay on a medical-surgical unit. The client has been having difficulty sleeping and appears depressed. Applying the holistic health model, which action taken by the nurse would be most appropriate? a. Ask the client's family to bring some items from home, such as a blanket, pillow, or pictures b. Reassure the client how lucky he or she is to have family visit often and reassure the client that he or she will be going home soon c. Suggest the client try some meditation and contact the health care provider to request medication to help with sleep d. Document the observations, and encourage the client that he or she will feel better once going home

a

An occupational therapist conducts daily relaxation exercises with clients who are receiving treatment on the psychiatric unit of a hospital. Stimulation of which of the following components of the nervous system will result in relaxation for the clients? a. Parasympathetic nervous system b. Autonomic nervous system c. Central nervous system d. Sympathetic nervous system

a

During a client interview, the client tells the nurse about using ginkgo biloba to improve memory. When reviewing the client's medication history, which medication if used by the client would be a cause for concern? a. Warfarin b. Acetaminophen c. Guaifenesin d. Azithromycin

a

The nurse explains to the client which statement is true regarding the difference between allopathic therapy and complementary and alternative therapy? a. Allopathic therapy emphasizes treatments for diseases. Complementary and alternative therapy emphasizes treatments for health. b. Allopathic therapy integrates mind and body. Complementary and alternative therapy separates mind and body. c. Allopathic therapy is the absence of illness. Complementary and alternative therapy states health is the imbalance of the body systems. d. Allopathic therapy uses herbs to treat. Complementary and alternative therapy uses medications to treat.

a

A nurse mentor is teaching a new nurse about the underlying beliefs of CHAs versus allopathic therapies. Which statements by the new nurse indicate that teaching was effective? Select all that apply. a) "CHA proponents believe the mind, body, and spirit are integrated and together influence health and illness." b) "CHA proponents believe that health is a balance of body systems: mental, social, and spiritual, as well as physical." c) "Allopathy proponents believe that the main cause of illness is an imbalance or disharmony in the body systems." d) "Curing according to CHA proponents seeks to destroy the invading organism or repair the affected part." e) "The emphasis is on disease for allopathic proponents and drugs, surgery, and radiation are key tools for curing." f) "According to CHA proponents, health is the absence of disease."

a, b, e. With CHA, mind, body, and spirit are integrated and together influence health and illness, and illness is a manifestation of imbalance or disharmony. Allopathic beliefs include: The main causes of illness are considered to be pathogens (bacteria or viruses) or biochemical imbalances, curing seeks to destroy the invading organism or repair the affected part, and emphasis is on disease and high technology. Drugs, surgery, and radiation are among the key tools for dealing with medical problems. According to allopathic beliefs, health is the absence of disease.

A client asks the nurse about taking herbal medications. Which of these is the nurse's best response regarding safety of the herbal medications? a. "Name brand products with herbal medications usually are of higher quality." b. "Your effects from herbal medications are quicker than with traditional medicine." c. "You can take combination products to save money on these medications." d. "Herbal medications are natural products so there are not side effects with these."

a. "Name brand products with herbal medications usually are of higher quality." Name brand products with herbal medications tend to be of higher quality so can be best to choose since the industry is not regulated by federal agencies. Herbal medications can have side effects, and many times work slower than traditional medications do, so clients should give them time to help and observe for any side effects. Single products are best, particularly if there are side effects; if there are combinations of medications, it is difficult to tell which caused a problem.

A nurse is caring for a hospitalized client who states: "I feel so sick all the time; my aura must be disturbed by all of these bad force fields." What is an appropriate NANDA-I diagnosis for this client? a. Disturbed energy field b. Social isolation c. Impaired coping d. Hopelessness

a. Disturbed energy field Defining characteristics of the disturbed energy field relate to movement, sounds, temperature change, visual changes, and disruption. The aura the client is referring to in this scenario is a vague, luminous glow surrounding something that may be an information-containing electromagnetic field, which can be likened to the data contained within a computer. Social isolation would refer to a client who is feeling isolated from others. Impaired coping refers to a client who is having difficulty dealing with a particular situation. Hopelessness refers to a client who feels there is no hope for improvement from an illness or in life in general.

The nursing instructor is discussing alternative therapy with a group of students. She explains that living organisms are "continuously connecting and interacting with their environment." Furthermore, the connecting and interacting signifies that the human body is a unified dynamic whole. The instructor is describing what theory to the students? a. Holism perspective b. Integrative perspective c. Allopathic perspective d. Medical system perspective

a. Holism perspective The instructor is describing the definition of holism. Integrative perspective is combining allopathic and complementary and alternative therapy (CAM). The medical system perspective is based on a philosophy and theory about health and illness along with specific types of treatment that may be based in culture. The allopathic perspective is based in biomedicine. Holism is the only perspective that is looking at parts interacting with each other and the environment.

Massage therapy is being used on clients during chemotherapy treatments. How does massage therapy help these patients? a. It relaxes muscles to increase circulation and release tension. b. It uses slow, gentle physical movements to cleanse the body. c. It physically moves joints into proper alignment to relieve stress. d. It uses pressure to balance and increase flow of energy.

a. It relaxes muscles to increase circulation and release tension. Massage therapy relaxes muscles to increase circulation and release tension. Acupressure uses pressure to balance and increase the flow of energy. Chi gong uses slow, gentle physical movements to cleanse the body. Chiropractic therapy physically moves joints into proper alignment to relieve stress.

A client's family member asks the nurse if he or she may bring essential oils for use in the hospital room for the client. What action should the nurse take? a. Provide education and determine if there is a sensitivity to the oils b. Inform the client and family that all essential oils are the same c. Tell the family member that essential oils are not proven to be effective treatment d. Refuse the use of essential oils in the client's room, because they may be a hazard

a. Provide education and determine if there is a sensitivity to the oils The essential oils of plants can be used as a therapy to improve physical, emotional, and spiritual well-being. The fragrance of these oils is believed to ultimately affect the very sensitive amygdala of the limbic system in the brain, where emotional memories are stored and released. Laboratory and animal studies have shown that certain oils have antibacterial, antiviral, antifungal, calming, and energizing effects. Commonly used essential oils in a health care setting are ginger or peppermint for nausea and lavender or chamomile for insomnia. Essential oils vary in quality and potency depending on the manufacturing processes. If specific essential oils are approved for use in an inpatient setting, the nurse provide education on these oils. Some people are highly sensitive to strong fragrances, particularly concentrated essential oils. This might preclude their use for specific clients.

A client at the clinic is considering the use of acupuncture to assist with smoking cessation. When describing this practice, the nurse would explain that which concept is involved as the focus? a. Qi b. Yang c. Yin d. Alignment

a. Qi Acupuncture addresses a person's qi. Qi (chi in Japanese) is believed to flow vertically in the body through an intricate structure of 12 primary meridians, energy circuits that nourish and support all cells and organs of the body Acupuncture either increases or decreases the flow of qi along the meridian, restoring the balance of yin and yang. Alignment is the focus of chiropractic health care.

The nurse is caring for a 48-year-old Native American/First Nations client hospitalized following a myocardial infarction. The client tells the nurse about not understanding the plan of care and asks to see the healer from home. The nurse arranges for the healer to come see the client. What primary benefit will the healer provide for the client? a. A healer can provide the client with a plan of care he or she understands and that is removed from traditional medicine based on the client's culture. b. Integrative health care that is part of the client's particular belief system can be developed by the healer that incorporates both traditional and complementary medicine. c. A healer can support the client in his or her beliefs that traditional healing practices may not be effective with the diagnosis and will create a plan of care based on the client's belief systems. d. Alternative, allopathic treatments including nutritional and lifestyle changes can be substituted by the healer to promote health and wellness.

b

When the nurse is obtaining a health history on an elderly client who has had a previous myocardial infarction, the daughter states, "I have been giving my father ginkgo biloba every day, as he is beginning to have some memory loss." How does the nurse respond to the daughter's statement? a. "Stop giving your father the herbal drug immediately." b. "How much of the herbal drug are you giving your father every day?" c. "Did you ask your health care provider before giving him the herbal drug?" d. "This may interfere with other medications your father is taking."

b

The client who is newly diagnosed with diabetes mellitus type 2 is concerned about eating products with sugar in them. What information does the nurse explain to the client regarding the use of sugar? a. Sugar increases natural bacterial flora in the GI tract. b. Excess sugar increases demand on the pancreas. c. Sugar assists with cellular absorption of nutrients. d. Healthy amounts of sugar are usually consumed.

b. Excess sugar increases demand on the pancreas. Excess sugar increases demand on the pancreas; therefore, less sugar should be consumed. The client who has diabetes mellitus type 2 may have a compromised pancreas. Further stress to the pancreas can cause further injury to the organ.

A nurse is providing care to a client who has come to the outclient clinic for chemotherapy. The client tells the nurse that to cope with the stress of chemotherapy, he uses a technique in which he "goes to my happy place, the beach, and I picture myself lying there under the warm sun, with the sound of the waves lapping at the shore." The nurse interprets this as which technique? a. Tai chi b. Guided imagery c. Yoga d. Meditation

b. Guided imagery Guided imagery focuses on evoking pleasant images to replace negative or stressful feelings and to promote relaxation. It involves using all five senses to imagine an event or body process unfolding according to a plan. During a painful or stressful event, the client can "go to a favorite place" and imagine being there with all the pleasant experiences related to that space. Meditation refers to a group of techniques in which the person learns to focus attention. Tai chi is a martial art, mind-body practice that involves physical movement, mental focus, deep breathing and relaxation. Yoga is a mind-body practice that involves the combination of physical movements, breathing practices, and relaxation techniques.

At a follow-up visit, a patient recovering from a myocardial infarction tells the nurse: "I feel like my life is out of control ever since I had the heart attack. I would like to sign up for yoga, but I don't think I'm strong enough to hold poses for long." What would be the nurse's best response? a) "Right now you should concentrate on relaxing and taking your blood pressure medicine regularly, instead of worrying about doing yoga." b) "There is a slower-paced yoga called Kripalu that focuses on coming into balance and relaxation that you could look into." c) "Ashtanga yoga is a gentle paced yoga that would help with your breathing and blood pressure." d) "Yoga is contraindicated for patients who have had a heart attack."

b. Kripalu, or "gentle yoga," focuses on relaxation and coming into balance. Ashtanga focuses on synchronizing breath with a fast-paced series of postures. The nurse should not discourage the use of yoga in patients who are healthy enough to participate. Yoga is not contraindicated in patients with controlled high blood pressure.

A nurse who has incorporated complementary and alternative medicine (CAM) into nursing practice is caring for a client in a short-term care facility. Which examples of nursing interventions are based on CAM? Select all that apply. a. The nurse schedules diagnostic tests for the client. b. The nurse investigates herbs that may stimulate the client's immune system. c. The nurse teaches the client how to meditate. d. The nurse uses guided imagery to relieve client anxiety. e. The nurse administers pain medication prescribed by the primary care provider. f. The nurse encourages the client to join a yoga class.

b. The nurse investigates herbs that may stimulate the client's immune system. f. The nurse encourages the client to join a yoga class. c. The nurse teaches the client how to meditate. d. The nurse uses guided imagery to relieve client anxiety. The term complementary and alternative medicine refers to interventions that are complementary therapies (CAM—they can be used with traditional medical interventions and thus complement them) as well as alternative modalities (not included in the scope of conventional medical care). Herbal remedies, yoga, meditation, and guided imagery are all forms of CAM. Administering pain medication and scheduling diagnostic tests are traditional allopathic (biomedicine) therapies.

A client asks the nurse for information about Traditional Chinese Medicine. The client specifically asks how Traditional Chinese Medicine might help manage one's chronic migraines. Which response by the nurse is best? a. "You might want to consider the use of botanicals, which can take the place of your prescribed medications." b. "Therapeutic Touch balances qi, which is part of vital life force, thus relaxing the muscles and increasing relief of migraine headaches." c. "Acupuncture uses fine needles inserted in the skin to stimulate natural painkillers and has proven to be beneficial in treating chronic migraines." d. "Meditation is the only alternative medicine that is recommended for chronic migraines."

c. "Acupuncture uses fine needles inserted in the skin to stimulate natural painkillers and has proven to be beneficial in treating chronic migraines." Diagnosis in Traditional Chinese Medicine is based in the balance, or lack thereof, of Yin and Yang. Yin and Yang are both aspects of qi, which is considered a vital life force. Acupuncture has been shown to be an effective way to manage chronic migraine headaches. Meditation is not the only therapy that can be used to treat chronic migraines, and Therapeutic Touch is not part of Traditional Chinese Medicine. The nurse should not recommend botanicals in place of prescribed medications.

When the nurse is obtaining a health history on an elderly client who has had a previous myocardial infarction, the daughter states, "I have been giving my father ginkgo biloba every day, as he is beginning to have some memory loss." How does the nurse respond to the daughter's statement? a. "Stop giving your father the herbal drug immediately." b. "This may interfere with other medications your father is taking." c. "How much of the herbal drug are you giving your father every day?" d. "Did you ask your health care provider before giving him the herbal drug?"

c. "How much of the herbal drug are you giving your father every day?" The nurse's response should be "How much of the herbal drug are you giving your father every day?" as this statement assesses the situation first. "Stop giving your father the herbal drug immediately" this statement is a directive statement and the nurse needs to use the nursing process to assess the situation. "Did you ask your health care provider before giving him the herbal drug" is accusatory and puts the client's family on the defensive side as they may not understand how the herbal drug works. "This may interfere with other medications your father is taking" is a true statement as the nurse understands that herbal medications can interfer with other medications. The nurse should default to assessing and then teaching if necessary.

A nurse is reading an online journal article about different approaches to health. The nurse is reading about a practice approach that is supported by evidence-based practice and is particularly effective when aggressive treatment is needed in an emergency situation. The nurse is reading about which type of approach? a. Traditional Chinese medicine b. Ayurveda c. Allopathic d. Naturopathy

c. Allopathic Allopathic medicine (or conventional medicine) is evidence-based practice that includes remarkable advances in biotechnology, surgical interventions, pharmaceutical approaches, and diagnostic tools. Allopathic medical care is particularly effective when aggressive treatment is needed in emergency or acute situations. Traditional Chinese medicine uses eight principles to analyze symptoms and categorize conditions, and uses the theory of five elements to explain how the body works. Naturopathy involves the belief that health is a dynamic state of being that provides abundant energy for people to deal with life in our complex society. Ayurveda involves the integration and balance of the body, mind and spirit.

Which principle does not encompass the basic goals of integrative medicine? a. Facilitate the body's innate healing abilities. b. Establish a partnership between client and practitioner. c. Reject allopathic medicine and embrace CAM practices. d. Focus on promoting health and preventing illness, as well as treating disease.

c. Reject allopathic medicine and embrace CAM practices. Integrative medicine is healing-oriented medicine that takes account of the whole person (body, mind, spirit, and community), including all aspects of lifestyle. It does not reject allopathic medicine and CAM practices. It includes establishing a partnership between client and practitioner, facilitating the body's innate healing abilities, and focusing on promoting health and preventing illness, as well as treating disease.

Which of the following forms of medicine combines health promotion, establishment of a partnership with the client and practitioner, and captures an evolving model of health care? a. homeopathic health care and medicine b. holistic health care and medicine c. integrative health care and medicine d. palliative health care and medicine

c. integrative health care and medicine Integrative medicine is the establishment of a partnership between the client and practitioner, and focuses on promoting health and preventing illnesses, as well as treating disease. Integrative health care more accurately reflects the cross-disciplinary reality and progressive acceptance of a broader aspect of care.

A client with stage IV colon cancer reports back pain and appears to be anxious. What response should the nurse provide when asked if therapeutic touch may be beneficial? a. "Therapeutic touch is based on the concept of qi and uses energy fields from the heavens and earth to restore a healthy state, but it will not cure your illness." b. "How often are you prepared to engage in therapeutic touch as it is an ongoing pain relief therapy?" c. "What do you currently know about therapeutic touch and why do you think this type of therapy will help?" d. "Therapeutic touch is a holistic practice that works to redirect energy in the body and may help with pain and anxiety."

d. "Therapeutic touch is a holistic practice that works to redirect energy in the body and may help with pain and anxiety." Therapeutic touch involves "unruffling," or clearing, congested areas of energy in the body and redirecting this energy. After assessing a client's "energy field," the nurse uses therapeutic touch to promote comfort, relaxation, healing, and a sense of well-being. The nurse should not dismiss or belittle the use of alternative therapies. Questioning the client's choice may appear dismissive or challenging which can cause the client to shut-down and not trust the nurse.

The nurse is educating a client on how to use herbs and supplements as part of an integrated treatment plan. Which teaching points would the nurse include? Select all that apply. a. Use the Internet to buy herbs and supplements. b. Be knowledgeable about the product and its therapeutic actions. c. Take a higher than recommended dose of herbs to initiate the therapeutic effect. d. Give the product adequate time to work. e. Whenever possible, buy products with more than one ingredient. f. Buy herbs and supplements that are standardized.

f. Buy herbs and supplements that are standardized. d. Give the product adequate time to work. b. Be knowledgeable about the product and its therapeutic actions. In the United States, FDA classifies herbs as a food and not a drug. This is important because herbs are not regulated for quality and potency as drugs are, and herbal formulas can be sold without being studied to see if they are helpful or harmful. For these reasons, herbs that have a certain degree of standardization should be purchased. There needs to be adequate time given for levels to accumulate in the client's system. Some herbs are either contraindicated or cause a synergistic effect when ingested with certain medications. The Internet is not a reliable source for purchasing herbal supplements. Products should not have more than one ingredient, for if the product contains several ingredients and the client has a reaction or positive response, it would be difficult to know which ingredient was the causative agent. Only the recommended dose should be ingested in order to prevent complications or toxicity.

dervived from substnaces that come from plants, minerals, or animals, such as red onion, arnica (mountain herb), crushed whole bees, white arsenic, poision ivy, belladonna (deadly nightshade) and stinging nettle. often formulated as sugar pellets to be placed under the tongue but may be in other forms such as ointments, gels, drops, creams, and tablets

homeopathic remedies

Scissor Gait

hypertonia in legs , hips and pelvis are flexed to appear patient is crouching and constant adduction of knees and thighs

Shamanism

illness and other forms of distress are thought to originate in the spirit world • the ability to access the spirit world

CAT: uses a variety of techniques designed to enhance the mind's ability to affect bodily function and symptoms

mind-body medicine

Humor and laughter

miscellanous therapy that brings out positive emotions and is highly personalized

Dystonia

muscle contractions in repetitive, twisting movements, involuntary in two or more parts of the body that is constant

The only healer is one

native american healing

holistic nursing

nursing practice built on a holistic philosophy

quadraplegia

paralysis of all four limbs legs, arms

Hemiplagia

paralysis of one side of the body

What are the main causes of illness in allopathic medicine?

pathogens

homeopathy

practice of medicine based on the belief of supporting the body while the symptoms are allowed to "run their course" to stimulate and strengthen the immune system and promote healing

acupuncture

procedure consisting of placing very thin, short, sterile needles at particular acupoints, believed to be centers of nerve and vascular tissue, along a meridian to either increase or decrease the flow of chi along the meridian, restoring the balance of yin and yang, and thereby contributing to healing

chiropractic health care

profession that focuses on the relationship between the body's structure—mainly the spine—and its functioning; its practitioners primarily perform adjustments (manipulations) to the spine or other parts of the body with the goal of correcting alignment problems, alleviating pain, improving function, and supporting the body's natural ability to heal itself

What are complementary health approaches?

refers to interventions that can be used with conventional medical interventions and thus complement them

Naturopathy empahsis on

responsibility, health maintenance, and disease prevention

On the advice of friends, a client on a palliative care unit has requested acupuncture. What it is the goal of this form of CAM?

restoring a healthy flow of energy along the meridians of the body

most widely practiced medical system on our planet

shamanism

Parkinson's disease

small. shuffling steps, with general slow movements and can lead to akinesia

Ayurveda

state of imbalance among body systems indian system of medicine

Qi Gong and Tai Chi is a

system of posture


संबंधित स्टडी सेट्स

Music Appreciation: Baroque Period

View Set

Chapter 07: The Nursing Process and Standards of Care for Psychiatric Mental Health Nursing

View Set

TSGTAH Was the Cold War Inevitable?

View Set

Mental Health Unit I: questions from Quizlet and NCLEX questions from Online Resources

View Set

68: Respiratory Failure and Acute Respiratory Distress Syndrome

View Set